Примеры решений матриц: Примеры решения матриц с ответами

Содержание

примеры с решением и объяснением

Матрицы представляют собой таблицы чисел, взаимосвязанных между собой. Над ними возможно проводить ряд разнообразных операций, о которых мы расскажем вам ниже.

Размер матрицы определяется её порядками — количеством строчек $m$ и столбцов $n$, которые в ней присутствуют. Строчки образованы элементами, стоящими на горизонтальных линиях, а столбцы — элементами, стоящими на прямых вертикальных линиях. В случае если количество строчек эквивалентно количеству столбцов — порядок рассматриваемой таблички определяется лишь одним значением $m = n$.

Замечание 1

Для любого элемента матрицы номер строчки, в которой он находится, записывается первым в индексе, а номер столбца — вторым, то есть запись $a_{ij}$ обозначает, что элемент стоит в $i$-ой строчке и в $j$-ом столбце.

Сложение и вычитание

Итак, о сложении и вычитании. Эти действия возможно проводить только с матрицами одинакового размера.

Для того чтобы осуществить эти действия, необходимо провести сложение или вычитание каждого элемента матрицы с элементом другой матрицы, стоящим на той же позиции, что элемент в первой.

В качестве примера найдём сумму $A+B$, где:

$A = \begin{pmatrix} a_{11} & a_{12} & a_{13} \\ a_{21} & a_{22} & a_{23}\\ a_{31} & a_{32} & a_{33} \\ \end{pmatrix}$

и $B = \begin{pmatrix} b_{11} & b_{12} & b_{13} \\ b_{21} & b_{22} & b_{23} \\ b_{31} & b_{32} & b_{33}\\ \end{pmatrix}$

Сумма любого элемента новой полученной матричной таблички $A + B$ равна $a_{ij} + b_{ij}$, например, элемент с индексом $11$ равен $a_{11} + b_{11}$,а весь результат целиком выглядит так:

$A + B = \begin{pmatrix} a_{11}+b_{11} & a_{12}+b_{12} & a_{13}+ b_{13} \\ a_{21}+ b_{21} & a_{22}+b_{22} & a_{23}+ b_{23} \\ a_{31}+ b_{31} & a_{32}+ b_{32} & a_{33} + b_{33} \\ \end{pmatrix}$

Вычитание для двух матриц $A-B$ осуществляется аналогично, но каждый элемент новой матрицы результата будет вычисляться по формуле $a_{ij} – b_{ij}$.

Обратите внимание, что сложение и вычитание для матриц возможно осуществлять только если их порядки одинаковые.

Пример 1

Решите следующие матричные примеры: $A + B$; $A – B$.

$A=\begin{pmatrix} 0 & 5 & 2 \\ 1 & -1 & 3 \\ -2 & 0 & 7 \\ \end{pmatrix}$

$B=\begin{pmatrix} 0 & 3 & 2 \\ -4 & 0 & -1 \\ 0 & 7 & -3 \\ \end{pmatrix}$

Объяснение:

Действия выполняем для каждой пары элементов $a_{ij}$ и $b_{ij}$ соответственно:

$A+B=\begin{pmatrix} 0+0 & 5+3 & 2+2 \\ 1-4 & -1+0 & 3 – 1\\ -2+0 & 0+7 & 7 – 3 \\ \end{pmatrix}=\begin{pmatrix} 0 & 8 & 4 \\ -3 & -1 & 2 \\ -2 & 7 & 4\\ \end{pmatrix}$

$A-B=\begin{pmatrix} 0-0 & 5-3 & 2-2 \\ 1+4 & -1-0 & 3 + 1\\ -2-0 & 0-7 & 7 + 3 \\ \end{pmatrix}=\begin{pmatrix} 0 & 2 & 0 \\ 5 & -1 & 4 \\ -2 & -7 & 10 \\ \end{pmatrix}$

Умножение матрицы на число

Для того чтобы произвести умножение матричной таблички на какое-либо число, нужно каждый её элемент умножить на это число, то есть любой элемент новой матрицы $C$, являющейся результатом произведения $A$ на $λ$ будет равен $с_{ij}=λ \cdot a_{ij}$.

Пример 2

Умножьте $A$ на $λ$, где $A=\begin{pmatrix} 1 & 0 & 2 \\ -1 & 3 & 0 \\ 2 & 1 & 3 \\ \end{pmatrix}$, а $λ=5$:

$A \cdot λ = 5 \cdot \begin{pmatrix} 1 & 0 & 2 \\ -1 & 3 & 0 \\ 2 & 1 & 3 \\ \end{pmatrix} = \begin{pmatrix} 1 \cdot 5 & 0 \cdot 5 & 2 \cdot 5 \\ -1 \cdot 5 & 3 \cdot 5 & 0 \cdot 5 \\ 2 \cdot 5 & 1\cdot 5 & 3\cdot 5 \\ \end{pmatrix} = \begin{pmatrix} 5 & 0 & 10 \\ -5 & 15 & 0 \\ 10 & 5 & 15 \\ \end{pmatrix}$.

Произведение матричных таблиц

Эта задача несколько сложнее предыдущих, но при этом в ней также нет ничего сложного.

Для осуществления умножения двух матриц $A \cdot B$ количество столбцов в $A$ должно совпадать с количеством строчек в $B$.

Математически это можно записать так:

$A_{m \times n}\cdot B_{n \times p} = С_{m \times p}$

То есть видя перемножаемые исходные матрицы можно сразу определить порядки получаемой новой. Например, если необходимо перемножить $A_{3 \times 2}$ и $B_{2 \times 3}$ — полученный результат будет иметь размер $3 \times 3$:

$\begin{pmatrix} a_{11} & a_{12} \\ a_{21} & a_{22} \\ a_{31} & a_{32} \\ \end{pmatrix} \times \begin{pmatrix} b_{11} & b_{12} &b_{13} \\ b_{21} & b_{22} & b_{23} \\ b_{31} & b_{32} &b_{33} \\ \end{pmatrix} = \begin{pmatrix} • & • & • \\ • & • & • \\ • & • & • \\ \end{pmatrix}= \begin{pmatrix} (a_{11}b_{11} + a_{12}b_{21}) & (a_{11}b_{12} + a_{12}b_{22}) & (a_{11}b_{13} + a_{12}b_{23}) \\ (a_{21}b_{11} + a_{22}b_{21}) & (a_{21}b_{12} + a_{22}b_{22}) & (a_{11}b_{13} + a_{22}b_{23}) \\ (a_{31}b_{11} + a_{32}b_{21}) & (a_{31}b_{12} + a_{32}b_{22}) & (a_{31}b_{13} + a_{32}b_{23}) \\ \end{pmatrix}$

Если число столбцов первого матричного множителя не совпадает с количеством строчек второго матричного множителя, то умножение выполнить невозможно.

Пример 3

Решите пример:

$A \times B = ?$, если $A=\begin{pmatrix} 1 & 0 & 2 \\ -1 & 3 & 0 \\ 2 & 1 & 3 \\ \end{pmatrix}$ и $B = \begin{pmatrix} 3 & – 1 & 2 \\ -4 & 0 & 2 \\ 1 & 1 & 2 \\ \end{pmatrix}$.

$A \times B = \begin{pmatrix} (1 \cdot 3 + 0 \cdot (-4) + 2 \cdot 1) & (1 \cdot(-1) + 0 \cdot 0 + 2 \cdot 1) & (1 \cdot 2 + 0 \cdot 2 + 2 \cdot 2) \\ (-1) \cdot 3 + 3 \cdot (-4) + 0 \cdot 1) & (-1 \cdot(-1) + 3 \cdot 0 + 0 \cdot 1) & (-1 \cdot 2 + 3 \cdot 2 + 0 \cdot 2) \\ (2 \cdot 3 + 1 \cdot (-4) + 3 \cdot 1) & 2 \cdot (-1) + 1 \cdot 0 + 3 \cdot 1) & (2 \cdot 2 + 1 \cdot 2 + 3 \cdot 2) \\ \end{pmatrix} $

$A \times B= \begin{pmatrix} (3 + 0+ 2) & (-1 + 0 + 2) & (2 + 0 + 4) \\ (-3-12+0) & (1 + 0 + 0) & (-2+6+0) \\ (6-4+3) & (-2 + 0 + 3) & (4 + 2 + 6) \\ \end{pmatrix} = \begin{pmatrix} 5 & 1 & 6 \\ -15 & 1 & 4 \\ 5 & 1 & 12 \\ \end{pmatrix}$.

Нахождение определителя матрицы

Определитель матрицы обозначается как $Δ$ или $\det$.

Замечание 2

Детерминант возможно найти только для квадратных разновидностей матриц.

В простейшем случае, когда матрица состоит из всего одного элемента, её определитель равен этому элементу: $det A = |a_{11}|= a_{11}$

Вычислить определитель от матрицы порядка двух можно следуя такому правилу:

Определение 1

Определитель матрицы размера 2 равен разности произведений элементов, стоящих на главной диагонали с произведением элементов с побочной диагонали:

$\begin{array}{|cc|} a_{11}& a_{12} \\ a_{21} & a_{22} \\ \end{array} = a_{11} \cdot a_{22} – a_{12} \cdot a_{21}$

В случае если определитель матрицы задан размером $3 \times 3$, то найти его можно используя мнемонические правила: Саррюса или треугольников, также можно разложить матрицу по строчке или столбцу или воспользоваться преобразованиями Гаусса.

{-1}$ на исходную матрицу даёт в результате единичную матрицу $E$.

Самый простой метод решения при поиске обратной матрицы — Жордана-Гаусса. Рядом с матрицей-подопытным кроликом записывается единичная того же размера, а затем исходная с помощью преобразований приводится к единичной, причём все выполняемые действия повторяются и с $E$.

Пример 4

Дана $A=\begin{pmatrix}{cc} 1& 2 \\ 3 & 4 \\ \end{pmatrix}$

Получить обратную матрицу.

Решение:

Пишем вместе $A$ и справа от неё соответствующего размера $E$:

$ \begin{array}{cc|cc} 1& 2 & 1& 0\\ 3 & 4& 0 & 1 \\ \end{array}$

Получаем нуль в последней строчке на первой позиции:прибавляем к ней верхнюю, умноженную на $-3$:

$ \begin{array}{cc|cc} 1& 2 & 1 & 0\\ 0 & -2 & -3 & 1 \\ \end{array}$

Теперь обнуляем последний элемент первой строчки. Для этого к верхней строчке плюсуем нижнюю:

$ \begin{array}{cc|cc} 1& 0 & -2 & 1\\ 0 & -2 & -3 & 1 \\ \end{array}$

Делим вторую на $-2$:

$ \begin{array}{cc|cc} 1& 0 & -2 & 1\\ 0 & 1& 3/2 & -1/2 \\ \end{array}$

Получили результат:

$A=\begin{pmatrix}{cc} -2& 1 \\ 3/2 & -1/2 \\ \end{pmatrix}$

Транспонирование матричных таблиц

Транспонирование — это смена строк и столбцов в матрице или определителе местами с сохранением их исходного порядка. T = 1 \cdot 5 \cdot (-3) + 4 \cdot (-2) \cdot 3 + (-1) \cdot 2 \cdot 6 – 4 \cdot 2 \cdot (-3) – 1 \cdot (-2) \cdot 6 – (- 1) \cdot 5 \cdot 3 = – 15 -24 – 12+24+12+15 = 0$.

Решение матричных уравнений: теория и примеры

  • Решение матричных уравнений: как это делается
  • Решение матричных уравнений: примеры

Матричные уравнения имеют прямую аналогию с простыми алгебраическими уравнениями, в которых присутствует операция умножения. Например,

ax=b,

где x – неизвестное.

А, поскольку мы уже умеем находить произведение матриц, то можем приступать к рассмотрению аналогичных уравнений с матрицами, в которых буквы – это матрицы.

Итак, матричным уравнением называется уравнение вида

A ⋅ X = B

или

X ⋅ A = B,

где

A и B – известные матрицы, X – неизвестная матрица, которую требуется найти.

Как решить матричное уравнение в первом случае? Для того, чтобы решить матричное уравнение вида A ⋅ X = B, обе его части следует умножить на обратную к A матрицу слева:

.

По определению обратной матрицы, произведение обратной матрицы на данную исходную матрицу равно единичной матрице: , поэтому

.

Так как E – единичная матрица, то E ⋅ X = X. В результате получим, что неизвестная матрица X равна произведению матрицы, обратной к матрице A, слева, на матрицу B:

.

Как решить матричное уравнение во втором случае? Если дано уравнение

X ⋅ A = B,

то есть такое, в котором в произведении неизвестной матрицы X и известной матрицы A матрица A находится справа, то нужно действовать аналогично, но меняя направление умножения на матрицу, обратную матрице A, и умножать матрицу B на неё справа:

,

,

.

Как видим, очень важно, с какой стороны умножать на обратную матрицу, так как . Обратная к A матрица умножается на матрицу B с той стороны, с которой матрица A умножается на неизвестную матрицу X. То есть с той стороны, где в произведении с неизвестной матрицей находится матрица A.

Как решить матричное уравнение в третьем случае? Встречаются случаи, когда в левой части уравнения неизвестная матрица X

находится в середине произведения трёх матриц. Тогда известную матрицу из правой части уравнения следует умножить слева на матрицу, обратную той, которая в упомянутом выше произведении трёх матриц была слева, и справа на матрицу, обратную той матрице, которая располагалась справа. Таким образом, решением матричного уравнения

A ⋅ X ⋅ B = C,

является

.

Пример 1. Решить матричное уравнение

.

Решение. Данное уравнение имеет вид

A ⋅ X = B, то есть в произведении матрицы A и неизвестной матрицы X матрица A находится слева. Поэтому решение следует искать в виде , то есть неизвестная матрица равна произведению матрицы B на матрицу, обратную матрице A слева. Найдём матрицу, обратную матрице A.

Сначала найдём определитель матрицы A:

.

Найдём алгебраические дополнения матрицы A:

.

Составим матрицу алгебраических дополнений:

.

Транспонируя матрицу алгебраических дополнений, находим матрицу, союзную с матрицей

A:

.

Теперь у нас есть всё, чтобы найти матрицу, обратную матрице A:

.

Наконец, находим неизвестную матрицу:

Решить матричное уравнение самостоятельно, а затем посмотреть решение

Пример 2. Решить матричное уравнение

.

Посмотреть правильное решение и ответ

.


Пример 3. Решить матричное уравнение

.

Решение. Данное уравнение имеет вид X ⋅ A = B, то есть в произведении матрицы A и неизвестной матрицы X матрица A находится справа. Поэтому решение следует искать в виде , то есть неизвестная матрица равна произведению матрицы B на матрицу, обратную матрице A справа. Найдём матрицу, обратную матрице A.

Сначала найдём определитель матрицы A:

.

Найдём алгебраические дополнения матрицы A:

.

Составим матрицу алгебраических дополнений:

.

Транспонируя матрицу алгебраических дополнений, находим матрицу, союзную с матрицей A:

.

Находим матрицу, обратную матрице A:

.

Находим неизвестную матрицу:

Нет времени вникать в решение? Можно заказать работу!

К началу страницы

Пройти тест по теме Матрицы

До сих пор мы решали уравнения с матрицами второго порядка, а теперь настала очередь матриц третьего порядка.

Пример 4. Решить матричное уравнение

.

Решение. Это уравнение первого вида: A ⋅ X = B, то есть в произведении матрицы A и неизвестной матрицы X матрица A находится слева. Поэтому решение следует искать в виде , то есть неизвестная матрица равна произведению матрицы B на матрицу, обратную матрице A слева. Найдём матрицу, обратную матрице A.

Сначала найдём определитель матрицы

A:

.

Найдём алгебраические дополнения матрицы A:

Составим матрицу алгебраических дополнений:

Транспонируя матрицу алгебраических дополнений, находим матрицу, союзную с матрицей A:

.

Находим матрицу, обратную матрице A, и делаем это легко, так как определитель матрицы A равен единице:

.

Находим неизвестную матрицу:

Пример 5. Решить матричное уравнение

.

Решение. Данное уравнение имеет вид X ⋅ A = B, то есть в произведении матрицы A и неизвестной матрицы X матрица A находится справа. Поэтому решение следует искать в виде , то есть неизвестная матрица равна произведению матрицы B на матрицу, обратную матрице A справа. Найдём матрицу, обратную матрице A.

Сначала найдём определитель матрицы A:

.

Найдём алгебраические дополнения матрицы A:

Составим матрицу алгебраических дополнений:

.

Транспонируя матрицу алгебраических дополнений, находим матрицу, союзную с матрицей A:

.

Находим матрицу, обратную матрице A:

.

Находим неизвестную матрицу:

Пример 6. Решить матричное уравнение

.

Решение. Данное уравнение имеет вид A ⋅ X ⋅ B = C, то есть неизвестная матрица X находится в середине произведения трёх матриц. Поэтому решение следует искать в виде . Найдём матрицу, обратную матрице A.

Сначала найдём определитель матрицы A:

.

Найдём алгебраические дополнения матрицы A:

.

Составим матрицу алгебраических дополнений:

.

Транспонируя матрицу алгебраических дополнений, находим матрицу, союзную с матрицей A:

.

Находим матрицу, обратную матрице A:

.

Найдём матрицу, обратную матрице B.

Сначала найдём определитель матрицы B:

.

Найдём алгебраические дополнения матрицы B:

Составим матрицу алгебраических дополнений матрицы B:

.

Транспонируя матрицу алгебраических дополнений, находим матрицу, союзную с матрицей B:

.

Находим матрицу, обратную матрице B:

.

Находим неизвестную матрицу:

НазадЛистатьВперёд>>>

Нет времени вникать в решение? Можно заказать работу!

К началу страницы

Пройти тест по теме Матрицы

Поделиться с друзьями

Начало темы “Матрицы”

Понятие матрицы

Продолжение темы “Матрицы”

Обратная матрица

Произведение двух матриц

Умножение матрицы на число

Сложение матриц

Другие темы линейной алгебры

Определители

Системы линейных уравнений

Примеры решения задач

1. Найти линейную комбинацию матриц , где,.

Решение:

.

2. Пусть – матрица размерности 2x 3, – матрица размерности 3 х 3. Найти произведенияи(если это возможно).

Решение: Используем формулу (2.1):

Произведение не существует, так как число столбцов матрицыB не совпадает с числом строк матрицы A: .

3. Найти, если.

Решение: .

.

4. Найти значение матричного многочлена , если , .

Решение: .

.

5. Транспонировать матрицу .

Решение: Так как у матрицы A две строки и три столбца, то у матрицы будет три строки и два столбца:.

6. Дана матрица . Найти обратную матрицу.

Решение: Воспользуемся первым способом нахождения обратной матрицы, т.е. формулой (2.2). Вычисляем определитель матрицы A:

.

Так как , то матрицасуществует. Найдем алгебраические дополнения ко всем элементам матрицыA:

; ;

; ;

; ;

;

;

.

Составим присоединенную матрицу: . Находим обратную матрицу, поделив каждый элемент присоединенной матрицы на определитель матрицы A. Получаем ответ:

.

7. Решить матричное уравнение: .

Решение: Запишем данное матричное уравнение в виде . Его решением является матрица (если существует матрица). Найдем определитель матрицыA: . Значит, обратная матрица существует, и исходное уравнение имеет (единственное) решение. Найдем обратную матрицу: , ; , . Найдем решение матричного уравнения:

.

8. Найти обратную к матрице , используя метод элементарных преобразований.

Решение: Припишем справа единичную матрицу

.

Разделив первую строку на три и обнулив элемент в первом столбце ниже тройки, получим

.

Умножив вторую строку на три и обнулив элемент во втором столбце выше , получим

.

Таким образом, .

Задачи для самостоятельного решения

1. Найти линейную комбинацию матриц , где

.

2. Найти произведения матриц и (если они существуют), где

.

3. Проверить коммутируют ли матрицы

и .

4. Найти значение матричного многочлена , еслии.

5. Вычислить произведение при заданной матрице .

6. Привести к ступенчатому виду матрицу .

7. Найти произведения матриц и, где

.

8. Найти обратную матрицу к матрице .

Решить матричные уравнения:

9. ;

10. .

11. Найти линейную комбинацию матриц , где

.

12. Найти произведения матриц и (если они существуют), где

.

13. Проверить, коммутируют ли матрицы и .

14. Найти значение матричного многочлена , если .

15. Вычислить произведение при заданной матрице.

16. Привести к ступенчатому виду матрицу .

17. Найти произведения матриц и, если

.

18. Найти обратную матрицу к матрице .

Решить матричные уравнения:

19. ;

20. .

Ответы:

1) ; 2) ;3) Да; 4) ;5) ; 6) ;7) ;8) ;9) ;10) ;11) ; 12) ;13) Нет; 14) ;15) ; 16) ;17) ;18) ;19) ;20) .

ПРАКТИЧЕСКОЕ ЗАНЯТИЕ 3

Решение систем линейных уравнений методом Крамера и Гаусса

1. Метод Крамера.

Система уравнений вида

(3.1)

называется системой m линейных уравнений с n неизвестными.

Коэффициенты этих уравнений записываются в виде матрицы А, называемой матрицей системы, а числа, стоящие в правой части системы, образуют столбец В, называемый столбцом свободных членов. Неизвестные системы так же записываются в столбец, называемый столбец неизвестных:

, ,

Используя произведение матриц, можно записать данную систему в матричном виде: .

Совокупность чисел называетсярешением системы, если каждое уравнение системы обращается в равенство после подстановки в него чисел вместо неизвестных.

Системы, не имеющие решения, называются несовместными.

Системы, имеющие решения, называются совместными. Заметим, что система может иметь единственное решение, а может иметь бесконечно много решений.

Для нахождения единственного решения систем с одинаковым количеством уравнений и неизвестных есть метод, называемый метод Крамера.

Система n уравнений с n неизвестными

имеет единственное решение, если определитель матрицы системы отличен от нуля. Это решение находится по формулам Крамера:

, (3.2)

где  – определитель матрицы системы, а k – определитель матрицы, полученной из матрицы системы заменой k-го столбца столбцом свободных членов.

Примеры решения задач типового расчёта по линейной алгебре и аналитической геометрии

Примеры решения задач типового расчёта

по линейной алгебре и аналитической геометрии.

Задача 1.1. Вычислить, представить ответ в алгебраической, тригонометрической, показательной формах.

Решение.  Представим комплексное число в тригонометрической форме. Для этого находится его модуль: , далее вычисляется аргумент комплексного числа: , либо , наконец,

, где , если комплексное число находится в первой или четвертой четверти, , если комплексное число находится во второй или третьей четверти. В данном случае аргумент комплексного числа равен: .

Поэтому тригонометрическая форма имеет вид: .

По формуле Муавра , где – аргумент комплексного числа , получаем:

,

окончательно: =.

В алгебраической форме: =,

в показательной форме:    =.

Задача 1.2. Вычислить .

Решение. Представим комплексное число  в тригонометрической форме:

=. Воспользуемся следующей формулой извлечения корня из комплексного числа : ), где=0,1,2,…,.

Т.е. корень -ой степени из комплексного числа принимает ровно  значений. Точки комплексной плоскости, соответствующие значениям , являются вершинами правильного многоугольника, вписанного в окружность радиуса  с центром в начале координат.

В данном примере получаем: =, где . Тогда корни принимают значения:

=(+),                         =(+),

=(+),                     =(+).

Для построения этих комплексных чисел на комплексной плоскости  проведем окружность радиуса . На окружности отметим точку =(+), далее, разбивая окружность на четыре равные части , изобразим остальные точки ,, . Заметим, что радиан соответствуют примерно.

Задача 2. Найти произведение матриц , где и .

Решение. ===

Задача 3. Найти матрицу, обратную данной. Сделать проверку. .

Решение. Находим определитель матрицы .

=, т.е. данная матрица является неособенной, обратная матрица существует. Вычислим соответствующие алгебраические дополнения :

==;           ==;          ;

;       ;          ;

;          ;        .

Находим обратную матрицу :

=. Проверка заключается в перемножении матриц

 (единичная матрица). Последнюю операцию выполнять аналогично примеру 2.

Задача 4. Решить систему линейных алгебраических уравнений  (СЛАУ) с помощью обратной матрицы: .

Решение. Найдем обратную матрицу к матрице системы .

Аналогично примеру 3 получаем =

Вектор – столбец решений находится по формуле:

=, или  Проверка осуществляется непосредственной подстановкой найденных значений неизвестных в исходную СЛАУ.

Задача 5. Найти ранг матрицы .

Решение. Последовательно осуществляем линейные преобразования строк данной матрицы для приведения ее к ступенчатому виду.

Шаг 1. Переставим в данной матрице первую и вторую строки.

Шаг 2. Умножим первый столбец на 1/2, четвертый столбец умножим на 1/5. В результате получим: .

Шаг 3. Умножим на 2 первую строку и прибавим её к третьей («заработаем» нуль на месте «3-1»,т.е. вместо (–2) получим (0)). В результате получим: .

Шаг 4. Умножим на –3 вторую строку и прибавим её к третьей («заработаем» нуль на месте «3-2»,т.е. вместо (–3) получим (0)). Получим:

Таким образом, получили ступенчатую матрицу эквивалентную данной, в которой две ненулевые строки, значит ее ранг равен 2: .

Задача 6. Решить систему методом Гаусса: .

Решение. Запишем расширенную матрицу  СЛАУ:

.

С помощью эквивалентных преобразований приведём матрицу к верхнетреугольному виду (прямой ход метода Гаусса).

Шаг1. Вычтем из второй строки первую, результат умножим на (–1/2):

Шаг 2. Вычитаем из третьей строки первую, умноженную на 2:

Шаг 3. Вычитаем из четвёртой строки первую, умноженную на 3:

Шаг 4. Прибавляем к третьей строке вторую, умноженную на 5, результат делим на 2:

Шаг 5. Вычитаем из четвёртой строки вторую:

Шаг 6. Прибавляем к четвёртой строке третью, умноженную на 6, результат делим на 17:

Теперь с помощью эквивалентных преобразований приведём матрицу к диагональному виду (обратный ход метода Гаусса).

Шаги 7, 8, 9. От третьей строки отнимаем четвёртую, умноженную на 2; от второй строки отнимаем четвёртую; от первой строки отнимаем четвёртую.

Шаги 10, 11. От первой строки отнимаем третью; от первой строки отнимаем вторую.

Теперь в последнем столбце получились искомые значения переменных, т.е.

Задача 7. Найти размерность и базис подпространства решений однородной системы линейных алгебраических уравнений (ОСЛАУ). .

Решение. Применим прямой ход метода Гаусса (приведем систему уравнений к верхнетреугольному виду).  Поступая аналогично примеру 6 вычтем из второй строки первую, умноженную на 3; вычтем из третьей строки первую, умноженную на 2; прибавим первую строку к четвёртой; получим:

,

последние три строки оказались одинаковыми, значит, матрицу можно привести к следующему виду:

.

В качестве базисных переменных можно взять , свободные – .

Ранг матрицы равен  2, т.е. , следовательно, размерность подпространства решений тоже равна 2.

Формируем фундаментальную систему решений (ФСР). Пусть переменные  принимают значения 1, 0 соответственно. Тогда из второго уравнения  находим значение  из первого уравнения находим значение  Тогда .

Пусть теперь переменные  принимают значения 0, 1 соответственно. Тогда, поступая аналогично, получим   Откуда .  – образуют ФСР, общее решение данной однородной СЛАУ имеет вид .

Задача 8. Доказать, что векторы  образуют базис и разложить вектор  по этому базису.

Решение. Ненулевые векторы образуют базис тогда и только тогда, когда они некомпланарны  (т.е. не лежат в одной плоскости), значит их смешанное произведение не должно быть равно 0:

, 121 + 333 + 221 – 223 – 231 – 131 = 12 ≠ 0. Векторы образуют базис.

лекции_1_курс_1_поток_осень_2018 | Кафедра высшей алгебры

Лекции по алгебре, 1 курс, 1 поток

Лектор: Д. А.Тимашёв

Лекции проходят по понедельникам на 1-й паре (9:00-10:35) в ауд. П4 на каждой чётной неделе и по средам на 2-й паре (10:45-12:20) в ауд. 16-10.

Литература
  1. А.И.Кострикин. Введение в алгебру. Часть I. Основы алгебры.

  2. Э.Б.Винберг. Курс алгебры.


3 сентября 2018
Лекция 1

Системы линейных уравнений (СЛУ) и их решения. Совместные и несовместные, определённые и неопределённые СЛУ. Матрица коэффициентов и расширенная матрица системы.

Элементарные преобразования систем линейных уравнений и их матриц. Элементарное преобразование приводит к эквивалентной СЛУ.

Метод Гаусса решения СЛУ: ведущие элементы (лидеры) строк матрицы, приведение к ступенчатому и улучшенному ступенчатому виду, ранг ступенчатой матрицы, анализ ступенчатой СЛУ, главные и свободные неизвестные, общее решение системы. n, пространство функций на множестве), простейшие следствия аксиом (единственность нулевого и противоположного векторов, умножение вектора на 0 и -1, умножение нулевого вектора на число).

Линейные комбинации векторов, их значения, тривиальная комбинация. Линейная зависимость, примеры (случай одного и двух векторов). Свойства линейной зависимости: сохранение линейной зависимости/независимости при увеличении/уменьшении системы векторов, эквивалентное определение (один из векторов системы линейно выражается через остальные), однозначное выражение вектора, добавление которого делает систему линейно зависимой. Основная лемма о линейной зависимости. Обобщение на бесконечные системы векторов.


12 сентября 2018
Лекция 3

Эквивалентность условий максимальности линейно независимой подсистемы в системе векторов и выражаемости остальных векторов системы через эту подсистему. Базис системы векторов: определение, примеры — базисы в пространствах геометрических векторов, стандартный базис в R^n. n), её геометрический смысл.


19 сентября 2018
Лекция 5

Линейные отображения векторных пространств: определение, геометрические примеры (поворот плоскости, проекция пространства на плоскость). Линейные отображения арифметических векторных пространств и их матрицы. Интерпретация СЛУ на языке линейных отображений.

Алгебраические операции над линейными отображениями и матрицами (сложение и умножение, умножение на числа). Матричная запись линейных отображений и СЛУ. Свойства матричных операций: коммутативность и ассоциативность сложения, ассоциативность и дистрибутивность умножения матриц на числа и между собой, некоммутативность умножения матриц, нулевая и противоположная матрицы, умножение на 0 и на 1. Векторное пространство матриц размера m×n.


26 сентября 2018
Лекция 6

Матричные операции и транспонирование.

Ранг произведения матриц.

Тождественное отображение и единичная матрица. Символы Кронекера. Обратная матрица: определение, единственность, связь с обратным линейным отображением. Произведение обратимых матриц обратимо. Невырожденные квадратные матрицы. Матрица обратима тогда и только тогда, когда она невырождена. Алгоритм нахождения обратной матрицы (с примером).

Элементарные матрицы, их основное свойство.


26 сентября 2018
Лекция 7

Обратимость элементарных матриц. Разложение произвольной невырожденной матрицы в произведение элементарных матриц.

Перестановки, их количество. Подстановки степени n, их двухрядная запись. Взаимно однозначное соответствие между перестановками и подстановками, количество подстановок степени n равно n!.

Умножение подстановок, его ассоциативность. Тождественная подстановка и подстановка, обратная к данной. Некоммутативность умножения подстановок.

Циклические подстановки (циклы), их орбиты, однорядная запись цикла. Независимость циклов, разложение произвольной подстановки в произведение независимых циклов, его единственность (с точностью до порядка сомножителей).


1 октября 2018
Лекция 8

Транспозиции, разложение произвольной подстановки в произведение транспозиций.

Инверсии в перестановке, чётность и знак перестановки и подстановки. Изменение чётности перестановки при транспозиции двух её элементов. Количество чётных и нечётных перестановок n элементов (или подстановок степени n) одинаково и равно n!/2. Определение знака подстановки по числу сомножителей в её разложении на транспозиции. Знак произведения подстановок. Знак обратной подстановки.

Определители квадратных матриц: определение по развёрнутой формуле. Свойства определителя как функции от набора строк матрицы: полилинейность, кососимметричность, определитель матрицы с нулевой строкой, с одинаковыми и с пропорциональными строками, неизменность при элементарных преобразованиях 1-го типа.

Определитель транспонированной матрицы, свойства определителя как функции от набора столбцов.

Определитель треугольной матрицы. Метод вычисления определителя приведением матрицы к треугольному виду.


3 октября 2018
Лекция 9

Определитель матрицы — единственная, с точностью до пропорциональности, полилинейная кососимметрическая функция от её строк.

Квадратная матрица невырождена тогда и только тогда, когда её определитель не равен 0.

Определитель матрицы с углом нулей. Определитель Вандермонда, его основное свойство. Определитель произведения матриц.

Миноры прямоугольной матрицы. Дополнительный минор и алгебраическое дополнение к элементу квадратной матрицы. Разложение определителя по строке и по столбцу. Лемма о фальшивом разложении определителя.


10 октября 2018
Лекция 10

Присоединённая матрица, её основное свойство. Формула для обратной матрицы. Пример: формула для обратной к матрице размера 2×2.

Правило Крамера для решения квадратных СЛУ.

Теорема о ранге матрицы (его совпадение с наибольшим порядком ненулевого минора), метод окаймляющих миноров для вычисления ранга матрицы.

Группы: определение, единственность нейтрального и обратного элемента. Абелевы группы. Примеры групп (в т.ч. симметрическая группа S_n).


15 октября 2018
Лекция 11

Сравнение мультипликативной и аддитивной терминологий в теории групп. Подгруппы: определение и примеры (в т.ч. знакопеременная группа A_n).

Кольца: определение, аддитивная группа кольца. Классы колец: ассоциативные, коммутативные, кольца с единицей. Примеры колец (кольцо целых чисел Z, кольцо квадратных матриц Mat_n, кольцо геометрических векторов в пространстве с векторным умножением). Простейшие следствия аксиом кольца (единственность 0, 1 и противоположного элемента, умножение на 0 и на -1).

Обратимые элементы в ассоциативных кольцах с единицей, примеры: 1 обратима, 0 не обратим в кольцах с числом элементов >1. Мультипликативная группа кольца, примеры (в т.ч. полная матричная группа GL_n).

Делители нуля, их необратимость. Возможность сокращения на множитель, не являющийся делителем нуля.

Поля: определение и примеры. Подкольца и подполя: определение и примеры (ZQR). Перенос теории линейных уравнений, векторов, матриц и определителей с поля R на произвольное поле.

Сравнимость целых чисел по модулю m, классы вычетов.


17 октября 2018
Лекция 12

Операции над вычетами, корректность их определения. Кольцо вычетов Z_m.

Делители нуля и обратимые элементы в кольцах вычетов. Z_m — поле тогда и только тогда, когда m — простое число.

Характеристика поля, примеры. Если характеристика поля положительна, то это простое число. Возведение суммы в степень, равную характеристике поля. Малая теорема Ферма.

Комплексные числа: аксиоматическое определение поля C, как минимального расширения поля R, содержащего квадратный корень из -1. Алгебраическая форма записи комплексных чисел: существование и единственность, действительная и мнимая части комплексного числа. +,•)). Единственность поля комплексных чисел с точностью до изоморфизма.

Геометрическая интерпретация комплексных чисел как точек или векторов на координатной плоскости. Геометрический смысл операций сложения и вычитания комплексных чисел. Модуль комплексного числа и сопряжённое число, их геометрический смысл, свойства операции сопряжения. Деление комплексных чисел в алгебраической форме.

Аргумент комплексного числа, его главная ветвь. Тригонометрическая форма записи комплексных чисел, её экспоненциальная версия. Свойства модуля и аргумента комплексного числа, умножение и деление комплексных чисел в тригонометрической форме, геометрический смысл этих операций, формула Муавра.

29 октября 2018
Лекция 14

Извлечение корней из комплексных чисел. Корни из единицы, первообразные корни.

Многочлены: неформальное определение, функциональная точка зрения на многочлены, её недостаток на примере поля вычетов Z_p (разные многочлены x и x^p задают одинаковые функции). ∞ — завершение построения модели.

Целостные кольца (области целостности). Кольцо многочленов над областью целостности целостно. Аддитивность степени многочлена. Обратимые элементы в кольце многочленов над областью целостности.

Многочлены над полем и полиномиальные функции. Задача о (полиномиальной) интерполяции. Теорема об интерполяции, интерполяционная формула Лагранжа. Число различных корней многочлена не превосходит его степени. Эквивалентность формального и функционального равенства многочленов над бесконечным полем.

Деление с остатком в кольце многочленов над полем (формулировка).


7 ноября 2018
Лекция 16

Доказательство теоремы о делении многочленов с остатком. Теорема Безу. Корни многочленов, кратность корня, простые и кратные корни. Число корней многочлена, с учётом их кратностей, не превосходит его степени.

Производная многочлена, её свойства. Высшие производные. Связь кратности корня со значениями производных.

Разложение многочлена по степеням линейного двучлена. Формула Тейлора.

Делимость в целостных кольцах. Ассоциированные элементы.


12 ноября 2018
Лекция 17

Наибольший общий делитель (НОД) двух элементов целостного кольца, его единственность с точностью до ассоциированности. Евклидовы кольца, примеры. Существование НОД в евклидовом кольце, алгоритм Евклида для его нахождения. Линейное выражение НОД через исходные элементы.

Простые элементы целостного кольца, примеры: простые числа и неприводимые многочлены. Разложение элемента евклидова кольца на простые множители, его единственность с точностью до перестановки множителей и ассоциированности. Факториальные кольца, факториальность евклидовых колец. Выяснение делимости элементов факториального кольца друг на друга, нахождение их НОД и НОК в терминах разложения на простые множители. Пример нефакториального целостного кольца.


14 ноября 2018
Лекция 18

Неприводимость многочлена зависит от поля коэффициентов. Многочлены 1-й степени неприводимы над любым полем. Неприводимые многочлены степени >1 не имеют корней (над данным полем).

Основная теорема алгебры комплексных чисел (ОТА): любой многочлен положительной степени над полем C имеет комплексный корень. Её следствия: неприводимые многочлены над полем C — это многочлены 1-й степени, разложение многочлена над полем C на линейные множители, число комплексных корней многочлена, с учётом кратностей, равно степени многочлена. Алгебраически замкнутые поля.

Основные понятия и факты математического анализа над полем C: ε-окрестности, пределы последовательностей и функций, непрерывность, существование минимума R-значной непрерывной функции на компакте. Лемма о неограниченном возрастании модуля многочлена. Лемма Д’Алабмера. Доказательство ОТА.

Комплексные корни многочлена с действительными коэффициентами, их разбиение на пары сопряжённых друг другу корней одинаковой кратности. Разложение многочлена над неприводимые множители (линейные и квадратичные с отрицательным дискриминантом) над полем R.


21 ноября 2018
Лекция 19

Проблема приближённого вычисления действительных (комплексных) корней многочлена сводится к нахождению количества корней в заданном интервале (заданной области). Теорема Декарта (правило знаков), оценка числа действительных корней (с учётом кратности) по одну сторону от заданной границы. Избавление от кратных корней (над полем характеристики 0).

Дроби над целостным кольцом как классы эквивалентности пар элементов кольца, определение алгебраических операций над дробями.


26 ноября 2018
Лекция 20

Свойства алгебраических операций над дробями, поле дробей целостного кольца.

Поле рациональных дробей K(x). Интерпретация рациональных дробей как функций.

Несократимые дроби. Представление элемента поля K(x) несократимой дробью, его единственность.

Правильные дроби. Представление рациональной дроби в виде суммы многочлена и правильной дроби, его единственность.

Простейшие дроби, их описание над полями C и R. Разложение правильной дроби в сумму простейших дробей.

Кольцо K[x_1,…,x_n] многочленов от нескольких переменных над ассоциативным коммутативным кольцом K с единицей: аксиоматическое определение, существование и единственность с точностью до изоморфизма (без доказательства).


28 ноября 2018
Лекция 21

Доказательство единственности кольца многочленов от нескольких переменных (с точностью до изоморфизма) и его существования (путём индуктивного построения).

Полиномиальные функции от нескольких переменных. Эквивалентность формального и функционального равенства многочленов от нескольких переменных над бесконечным полем.

Степень одночлена и многочлена (полная и по отдельным переменным), однородные многочлены, разложение многочлена в сумму однородных компонент.

Лексикографический порядок на одночленах, его свойства. Старший член ненулевого многочлена. Старший член произведения многочленов над целостным кольцом K, целостность кольца K[x_1,…,x_n].

Многочлены от одной переменной над факториальным кольцом A, примитивные многочлены, лемма Гаусса. Факториальность кольца многочленов A[x], факториальность колец Z[x] и K[x_1,…,x_n], где K — поле.


10 декабря 2018
Лекция 22

Симметрические многочлены: определение и примеры. Степенные суммы и элементарные симметрические многочлены.

Теорема Виета: выражение значений элементарных симметрических многочленов на корнях многочлена от одной переменной через его коэффициенты.

Основная теорема о симметрических многочленах: существование и единственность выражения произвольного симметрического многочлена через элементарные симметрические многочлены.

Выражение значения симметрического многочлена на корнях многочлена от одной переменной через его коэффициенты.

Дискриминант многочлена от одной переменной, его основное свойство: дискриминант равен 0 тогда и только тогда, когда многочлен имеет кратные корни.


12 декабря 2018
Лекция 23

Вычисление дискриминанта через определитель из степенных сумм корней многочлена. Инвариантность дискриминанта относительно сдвига переменной на константу, сведение к дискриминанту неполного многочлена. Дискриминант неполного кубического трёхчлена, его связь с количеством вещественных корней.

Результант двух многочленов от одной переменной, его свойства, вычисление результанта через определитель из коэффициентов многочленов. Связь дискриминанта многочлена c результантом многочлена и его производной.


19 декабря 2018
Лекция 24

Возведение элемента группы в целую степень, свойства степени.

Порядок элемента группы, его свойства. Пример: порядок подстановки.

Циклическая подгруппа, порождённая элементом группы, циклические группы, примеры: аддитивные группы колец Z и Z_m. n=e в группе порядка n. Теорема Эйлера о вычетах.

О новом простом методе снижения высокой размерности данных / Хабр

О новом методе решения проблемы оценки ковариационной матрицы в данных высокой размерности [научная работа опубликована в 2012 году] рассказываем к старту нашего флагманского курса по Data Science. Подробности — под катом:

  • Оптимальная оценка ковариационной матрицы в больших размерностях
  • Основы
  • Проблема
  • Решение
  • Пример кода
  • Заключение
  • Литература


Оптимальная оценка ковариационной матрицы в больших размерностях


Приложения в области статистики, машинного обучения и даже в таких областях, как финансы и биология, часто нуждаются в точной оценке ковариационной матрицы. Однако в настоящее время многие из этих приложений используют данные высокой размерности, и поэтому обычная (выборочная) ковариационная оценка просто не подходит. Таким образом, множество работ за последние два десятилетия пытались решить именно эту проблему. Чрезвычайно мощный метод, появившийся в результате этих исследований, — «нелинейное снижение размерности» [1]. Я сосредоточусь на новейшей версии этого подхода, квадратичном обратном линейном снижении размерности (QIS) от [2]. Этот подход делает нелинейное снижение размерности простым в реализации и быстрым для вычисления.



А на наших курсах вы научитесь применять математику в решении конкретных проблем бизнеса:
  • Профессия Data Scientist (24 месяца)
  • Профессия Data Analyst (18 месяцев)

Давайте начнём с основ (если вы уже имели дело с ковариационными матрицами, то можете смело пропустить этот раздел).

Основы


Ковариационная матрица содержит дисперсии и ковариации двух или более случайных величин. То есть, если мы посмотрим на случайные величины , и , то ковариационная матрица будет иметь вид:

Поскольку ковариация между и и и одинакова, матрица обязательно будет симметричной. Более того, из линейной алгебры мы знаем, что эта матрица имеет так называемое разложение на собственные значения, т. е.

Важной частью здесь является лямбда-матрица, которая представляет собой диагональную матрицу, содержащую собственные значения ковариационной матрицы

Таким образом, мы можем диагонализовать ковариационную матрицу, а её диагональные элементы будут содержать важную информацию. Например, они сразу сообщают нам ранг ковариационной матрицы: то есть, просто количество ненулевых собственных значений. Если матрица имеет полный ранг, т.е. все её собственные значения больше нуля, это означает, что случайные величины полностью разбросаны в -мерном пространстве. Это также очень полезно, потому что

В частности, мы видим, что ковариационная матрица обратима тогда и только тогда, когда все её собственные значения отличны от нуля или если ранг матрицы равен .

Каждая симметричная матрица может быть разложена на произведение ортогональных матриц и (вещественной) диагональной матрицы. Значения диагональной матрицы раскрывают важные свойства и могут использоваться для лёгкого вычисления обратной матрицы, если она существует.

Проблема


Предположим, что теперь у нас есть выборка из независимых одинаково распределённых случайных векторов в -измерениях с ожидаемым значением, равным нулю (для простоты). Ковариационная матрица может оцениваться с помощью обычной выборочной ковариационной матрицы (далее — ВКМ):

Это оказывается как раз оценкой максимального правдоподобия, если распределение ваших данных соответствует распределению Гаусса. Такая оценка обладает всевозможными благоприятными свойствами и, в частности, известно, что она сходится к истине, когда стремится к бесконечности, а остаётся постоянным (или медленно растёт относительно так, что стремится к нулю).

Однако обратите внимание, что в приведённой выше матрице у нас есть элементы для оценки. Здесь — это количество способов выбрать два элемента из , которое мы делим на 2, потому что матрица симметрична. В выборке это становится проблемой, если не намного больше, чем ! Это легче всего увидеть, когда оказывается больше . В этом случае можно показать, что ВКМ матрица имеет ранг не более чем . В частности, даже если истинная ковариационная матрица обратима (имеет все собственные значения больше нуля), ВКМ будет иметь нулевых собственных значений и, таким образом, никогда не будет обратимой. Поэтому, если вам нужна оценка обратной ковариационной матрицы, как в линейном дискриминантном анализе или при вычислении значения гауссовой плотности, это может оказаться проблемой.

Это может показаться крайностью, но даже если у вас есть, скажем, , ошибка оценки ВКМ может быть существенной. Давайте теперь посмотрим на удачный способ решения этой проблемы.

Решение


Тут я буду радикален. Конечно, у этой проблемы есть множество решений, и в зависимости от ситуации некоторые из них работают лучше других. Даже простое изменение того, как вы измеряете успех, может изменить порядок методов.

Возможно, простая идея, которая привела к множеству исследований и появлению новых приложений, состоит в том, чтобы просто взять линейную комбинацию ВКМ и единичной матрицы:

Интенсивность снижения размерности может быть выбрана различными способами, но чаще всего она автоматически выбирается по данным из теоретических соображений. Если рассматривать разложение на собственные значения тождества и ВКМ, то мы уже видим нечто своеобразное:

Получается, у нас просто есть новые собственные значения, которые являются выпуклой комбинацией ВКМ и . Таким образом, если -тое собственное значение было равно 2, то регуляризованное значение теперь равно . В частности, поскольку наименьшее собственное значение ВКМ равно 0, наименьшее регуляризованное собственное значение теперь равно . Итак, пока , регуляризованная матрица всегда будет обратимой! Формула также может быть знакома по L2-регуляризованной регрессии:

Принцип, согласно которому собственные векторы остаются неизменными, а собственные значения адаптируются, является важным в этих методах уменьшения размерности. Это имеет смысл, потому что, как правило, бесполезно правильно оценивать все параметры, когда близко или превышает . Но собственные значения — это совсем другая история, и они могут быть достижимы. Это основная идея нелинейного уменьшения размерности, из-за этой идеи оно на шаг впереди.

Оценка собственных векторов в большой размерности бесполезна. Таким образом, важным принципом является адаптация собственных значений и просто сохранение собственных векторов неизменными.

В приведённом выше линейном уменьшении размерности, если выбрать «правильно», то можно фактически достичь оптимальной производительности в асимптотическом пределе, когда и , и идут вместе до бесконечности. Это было установлено в статье 2004 г. [3] и показывает оптимальную оценку собственного значения, если класс оценок является линейным, как в примере выше. То есть класс линейных оценок снижения размерности — это просто все оценки вида (1) при изменении .

Нелинейное уменьшение размерности даёт асимптотическую оценку в гораздо более крупном классе, который не обязательно должен быть просто линейной функцией ВКМ. В частности, решает проблему

где — некоторая функция потерь. Мы ищем выбор (уменьшенных) собственных значений, который максимально приближает результирующую матрицу к истинной ковариационной матрице, если нам разрешено использовать только собственные векторы ВКМ! Как мы видели ранее, линейное уменьшение размерности является частным случаем этого, потому что мы также меняем только собственные значения. Однако теперь способ, которым мы определяем лямбду, не ограничивается линейностью, что обеспечивает гораздо большую гибкость!

Интересно, что (недостижимое) оптимальное решение вышеизложенного вполне интуитивно понятно:

где — это просто примеры собственных векторов из предыдущего:

То есть лучшим решением для собственных значений в этом контексте являются не истинные значения, а то значение, которое мы получаем, когда применяем собственные векторы ВКМ к истинной ковариационной матрице.

Лучшее, что мы можем сделать для диапазона функций потерь, — это оценить значения, которые получаются, когда выборки собственных векторов применяются к истинной ковариационной матрице.

Оказывается, метод нелинейного снижения размерности способен последовательно оценивать эти оптимальные элементы, когда и вместе стремятся к бесконечности! Давайте теперь посмотрим на пример.

Пример кода


Итак, нелинейное снижение размерности красиво выглядит в теории, но применимо ли оно на практике? Вот тут-то и появляются две совсем недавние статьи, благодаря которым нелинейное снижение размерности превратилось из причудливой идеи, требующей больших вычислений, в реально используемый инструмент. В частности, метод QIS (квадратичное обратное линейное снижение размерности, [2]) можно рассчитать в несколько строк кода! Но вам даже этого делать не нужно, так как весь необходимый код [Python, R, MatLab] доступен здесь.

Давайте рассмотрим приложение в R, использующее функцию qis. В этом примере мы берём сложную истинную ковариационную матрицу, которая имеет

Так, в частности, дисперсия равна 1, и чем дальше друг от друга индексы и , тем меньше ковариация между ними. {abs(i-j)} ) } ) # Take the eigendecomposition of the true covariance matrix spectral<-eigen(Sig) # Simulate data X<-rmvnorm(n = n, sigma = Sig) # Take the eigendecomposition of the scm samplespectral<-eigen(cov(X)) # Use QIS and take the eigendecomposition Cov_qis <- qis(X) qisspectral<-eigen(Cov_qis) # Rename qisspectral$U<-qisspectral$vectors qisspectral$Lambda<-qisspectral$values # Want u_j’*Sig*u_j for all j=1,…,p whatwewant<-diag( t(qisspectral$U)%*%Sig%*%qisspectral$U ) #check on first value whether its really calculated correctly (whatwewant[1]-t(qisspectral$U[,1,drop=F])%*%Sig%*%qisspectral$U[,1,drop=F])

Код моделирует данные из 1000-мерной нормали и вычисляет выборочную ковариационную матрицу с её собственными значениями. Обратите внимание, что это работает без проблем, несмотря на то, что . Это одна из опасностей ВКМ, она работает даже тогда, когда это может быть неуместно. Затем вычисляется матрица QIS вместе с собственными значениями и собственными векторами (которые совпадают с собственными векторами ВКМ). Наконец, мы вычисляем теоретическое оптимальное решение, рассмотренное выше.

Давайте построим график:

plot(sort(samplespectral$values, decreasing=T), type="l", cex=0.8, lwd=1.5, lty=1)
lines(sort(spectral$values, decreasing=T), type="l", col="darkblue", cex=0.8, lwd=1.5, lty=2)
lines(sort(whatwewant, decreasing=T), type="l", col="darkred", cex=0.8, lwd=1.5,, lty=3)
lines(sort(qisspectral$Lambda, decreasing=T), type="l", col="darkgreen", cex=0.8, lwd=1.5, lty=4)
legend(500, 20, legend=c("Sample Eigenvalues", "True Eigenvalues", "Attainable Truth", "QIS"),
       col=c("black", "darkblue", "darkred", "darkgreen"), lty=1:4, cex=0.8)

Что даёт

Чем интересен этот график? Во-первых, мы видим, что выборочные собственные значения сильно отличаются — они оказываются выше самых больших собственных значений и ниже самых маленьких. В частности, последние 1000 – 800 = 200 собственных значений равны нулю. Эта «сверхдисперсия» хорошо известна в больших размерностях: малые собственные значения оцениваются как слишком маленькие, а большие — как слишком большие. С другой стороны, мы видим, что нелинейно уменьшенные собственные значения (выделены зелёным) довольно близки к истинным значениям, показанным синим. Что ещё более важно, они очень близки к красной линии, которая является достижимой истиной выше, то есть !

Действительно, тогда оценка общей матрицы лучше более чем на 30%:

((norm(cov(X)-Sig,type="F")-norm(Cov_qis-Sig, type="F"))/norm(Cov_qis-Sig, type="F"))
[1] 0.3088145

Эта разница может быть намного больше в зависимости от формы истинной базовой ковариационной матрицы. Важно отметить, что QIS работает примерно так же, как и обычная оценка ковариационной матрицы, когда очень мало по сравнению с , и становится (намного) лучше, как только растёт относительно . Таким образом, если достаточно велико, то есть , может быть даже выгодно всегда использовать QIS напрямую!

Заключение


В этой статье даётся представление о методе нелинейного снижения размерности выборочной ковариационной матрицы. После более концептуального/математического введения, небольшой пример кода иллюстрирует метод в R. Код для метода также доступен в Matlab и Python.

Есть много реальных приложений, где широко используется этот метод, в частности, в области финансов. Знаете ли вы другие приложения, где это может быть полезно, из вашего собственного рабочего опыта? Я всегда рад услышать об интересных вариантах использования и наборах данных.

Литература


[1] Ledoit, O. and Wolf, M. (2012). Nonlinear Shrinkage Estimation of Large-Dimensional Covariance Matrices. The Annals of Statistics, 40(2):1024–1060.

[2] Ledoit, O. and Wolf, M. (2022). Quadratic Shrinkage for Large Covariance Matrices. Bernoulli, 28(3):1519–1547.

[3] Ledoit, O and Wolf, M (2004). A Well-Conditioned Estimator for Large-Dimensional Covariance Matrices. Journal of Multivariate Analysis, 88(2):365–411.



А мы поможем прокачать ваши навыки или с самого начала освоить профессию, востребованную в любое время:
  • Профессия Data Scientist (24 месяца)
  • Профессия Data Analyst (18 месяцев)
  • Профессия Fullstack-разработчик на Python (15 месяцев)

Матрицы и определители: задачи с решениями

Задача 1

Каковы размеры матрицы $A$?
$А=\влево[ \begin{массив}{ccccc} 2 и -2 и 0 и 1 и 1 \\ 0 и 1 и 1 и 0 и 3 \\ 1 и -1 и 3 и 0 и 1 \\ 1 и 1 и 1 и 1 и 1 \конец{массив} \right] $

5

$5 \times 4$

$4 \times 5$

20

Задача 2

$A=\left[ \begin{массив}{ccccc} 2 и -2 и 0 и 1 и 1 \\ 0 и 1 и 1 и 0 и 3 \\ 1 и -1 и 3 и 0 и 1 \\ 1 и 1 и 1 и 1 и 1% \конец{массив}% \справа] $

Какой элемент $A_{2,4}$?

Задача 3

$A=\left[ \begin{массив}{ccccc} 2 и -2 и 0 и 1 и 1 \\ 0 и 1 и 1 и 0 и 3 \\ 1 и -1 и 3 и 0 и 1 \\ 1 и 1 и 1 и 1 и 1% \конец{массив}% \right] $

Какой элемент $A_{3,2}$?

Задача 4 прислал Oyoo Fredrick Ochieng

Дайте определение термину единичная матрица (единичная матрица).

Все элементы едины.

Единицы на одной из диагоналей и нули в других местах.

Единицы в первой строке и первом столбце и нули в остальных местах.

Единицы на главной диагонали и нули в остальных местах.

Задача 5

Запишите следующую систему уравнений в виде расширенной матрицы.

$\слева\{ \начать{массив}{с} 3х-2у=3\\ 5х+у=0 \конец{массив} \right\} $

$\left[ \begin{array}{ccc} 3 & -2 \\ 5 & 1 \end{array} \right]$

$\left[ \begin{matrix} 3 & 5 \\ -2 & 1 \\ 3 & 0 \end{matrix} \right]$

$\left[ \begin{array}{ccc} 3 & -2 & 3 \\ 5 & 1 & 0 \end {массив} \right]$

$\left[ \begin{array}{ccc} 3 & -2 & 3 \\ 5 & 1 & 3 \end{array} \right]$

Задача 6

Какова сумма матриц?
$\влево[ \begin{массив}{cc} 2 & -1 \\ 1 и 3 \конец{массив} \вправо] +\влево[ \begin{массив}{cc} 1 и 0 \\ 2 и -1 \конец{массив} \right] =$

$\left[ \begin{array}{cc} 3 & 1 \\ 2 & 2 \end{array} \right]$

$\left[ \begin{array}{cc} 3 & -1 \\ 3 & 2 \end{array} \right]$

$\left[ \begin{array}{cc} 3 & -1 \\ 3 & 4 \end{array} \right]$

$\left[ \begin{array}{cc} 3 & 3 \\ 2 & -1 \end{array} \right]$

Задача 7

Найдите матрицу $A$ так, чтобы выполнялось следующее равенство.

$A+\влево[ \begin{массив}{cc} 2 и 3 \\ -4 и 1 \конец{массив} \вправо] =\влево[ \begin{массив}{cc} 5&-1\\ 1 и 5 \конец{массив} \right] $

$A=\left[ \begin{array}{cc} 5 & 4 \\ 3 & -4 \end{array} \right]$

$A=\left[ \begin{array }{cc} 3 & -4 \\ 5 & 4 \end{массив} \right]$

$A=\left[ \begin{array}{cc} -3 & 4 \\ -5 & -2 \end{array} \right]$

$A=\left[ \begin{array}{ cc} 7 и 2 \\ -3 и 4 \end{массив} \right]$

Задача 8

Что получится в результате умножения?
$5 \раз\влево[ \начать{массив}{с} -2\ 3\\ -4 \конец{массив} \right] =$

$\left[ \begin{array}{ccc} -20 & 15 & -10 \end{array} \right]$

$\left[ \begin{array}{c} 10 \\ 15 \\ 20 \end{array} \right]$

$\left[ \begin{array}{c} -20 \\ 15 \\ -10 \end{array} \right]$

$\left[ \begin{array}{c} -10 \\ 15 \\ -20 \end{array} \right]$

Задача 9

Найдите матрицу $X$.

$\frac{3}{2}X+\left[ \begin{массив}{cc} -1 и 3 \\ 2 и -2 \конец{массив} \вправо] =\влево[ \begin{массив}{cc} 3 и -4 \\ 5 и 4 \конец{массив} \right] $

$X=\left[ \begin{array}{cc} 2 & 4 \\ \frac{8}{3} & -\frac{14}{3} \end{array} \right ]$

$X=\left[ \begin{array}{cc} 6 & -\frac{21}{2} \\ \frac{9}{2} & 9 \end{array} \right]$

$X=\left[ \begin{array}{cc} \frac{8}{3} & -\frac{14}{3} \\ 2 & 4 \end{array} \right]$

$X=\left[ \begin{array}{cc} 3 & -\frac{3}{2} \\ \frac{21}{2} & 3 \end{array} \right]$

Задача 10

Если $A=B\times C$, найти матрицу $A$.

$B=\слева[ \begin{массив}{ccc} 1&-3&-2\ 2 и 0 и 1 \конец{массив} \right]$      $C=\left[ \begin{массив}{cc} 2 и 1 \\ -2 и -1 \\ 3 и 0 \конец{массив} \right]$

$\left[ \begin{array}{cc} 4 & 1 \\ 0 & 3 \\ 3 & 0 \end{array} \right]$

$\left[ \begin{array}{ccc} 2 & 6 & -6 \\ 2 & 0 & 0 \end{array} \right]$

$\left[ \begin{array}{cc} 7 & 2 \\ 2 & 4 \end{array} \right]$

$\left[ \begin{array}{cc} 2 & 4 \\ 7 & 2 \end{array} \right]$


Задача 11

Найдите определитель матрицы.
$А=\влево[ \begin{массив}{cc} 2 и -3 \\ 4 и 5 \конец{массив} \справа] $

Задача 12

Найдите определитель матрицы.
$А=\влево[ \begin{массив}{cc} 3 и 4 \\ 0 и 0 \конец{массив} \справа] $

Задача 13

Найти обратную матрицу $A=\left[ \begin{массив}{cc} 2 и -3 \\ 4 и 5 \конец{массив} \right] $

$\left[\begin{matrix}110 & 66\\-88 & 44\end{matrix} \right]$

$\left[\begin{matrix}5 & 3\\ – 4 & 2\end{matrix} \right]$

$\frac{1}{22}$

$\left[\begin{matrix}\frac{5}{22} & \frac{3}{ 22}\\\frac{-2}{11} & \frac{1}{11}\end{matrix} \right]$ 9{-1}=\left[\begin{array}{cc} 0 & 0 \\ 0 & 0 \end{array}\right]$

Обратное для A не существует.

Задача 16

$A=\left[ \begin{массив}{cc} 7 и -4 \\ 4 и -3 \конец{массив} \вправо]$ $B=\влево[ \begin{массив}{cc} \frac{3}{5} & -\frac{4}{5} \\ \frac{4}{5} & -\frac{7}{5} \конец{массив} \right] $
Являются ли $A$ и $B$ мультипликативно обратными (можно ли написать, что $A \cdot B = B \cdot A$)?

Задача 17

$A=\left[ \begin{массив}{cc} 2 и -3 \\ 1 и -2 \конец{массив} \right]$   $B=\left[ \begin{массив}{cc} -2 и 1 \\ -3 и 2% \конец{массив} \справа]$
Являются ли $A$ и $B$ мультипликативно обратными (можно ли написать, что $A \cdot B = B \cdot A$)?

Задача 18

$A=\left[ \begin{массив}{cc} 8 и 9 \\ -1 и 2 \конец{массив} \right]$   $B=\left[ \begin{массив}{cc} \frac{2}{25} & -\frac{1}{5} \\ \фракция{3}{25} и \фракция{9}{25} \конец{массив} \right]$
Являются ли $A$ и $B$ мультипликативно обратными?

Задача 19

$A=\left[ \begin{массив}{cc} 8 и 9 \\ -1 и 2 \конец{массив} \right]$   $ B=\left[ \begin{массив}{cc} \frac{2}{25} & -\frac{9{25} \\ \фракция{1}{25} и \фракция{8}{25} \конец{массив} \right]$
Являются ли $A$ и $B$ мультипликативно обратными?

Задача 20

Какое значение должно быть у $x$, чтобы $B$ было обратным $A$?
$А=\влево[ \begin{массив}{cc} 1 и 3 \\ -1 и 2 \конец{массив}% \вправо] \qquad B=\влево[ \begin{массив}{cc} \frac{2}{5} & x \\ \фракция{1}{5} и \фракция{1}{5} \конец{массив} \право]$

Задача 21

При каком значении $x$ матрица $A$ не имеет обратной?

$А=\влево[ \begin{массив}{cc} 2 и 3 \\ х & -2 \конец{массив} \right] $

$\frac{4}{3}$

$-\frac{3}{4}$

$-\frac{4}{3}$

$\frac{3} {4}$

Задача 22

Какое значение должно быть у $x$, чтобы матрица $A$ не имела обратной?

$А=\слева[ \begin{массив}{cc} 1 и 2+х \\ х & -1 \конец{массив} \право]$

Сообщите о проблеме на этой странице.

Правильно:

Неправильно:

Нерешенные проблемы:

4.6: Решение систем уравнений с использованием матриц

  1. Последнее обновление
  2. Сохранить как PDF
  • Идентификатор страницы
    5142
    • OpenStax
    • OpenStax
    Цели обучения

    К концу этого раздела вы сможете:

    • Записывать расширенную матрицу для системы уравнений
    • Использование строковых операций над матрицей
    • Решение систем уравнений с использованием матриц

    Прежде чем начать, пройдите этот тест на готовность.

    1. Решите: \(3(x+2)+4=4(2x−1)+9\).
      Если вы пропустили эту проблему, просмотрите [ссылка] .
    2. Решите: \(0,25p+0,25(x+4)=5,20\). 92\).
      Если вы пропустили эту проблему, просмотрите [ссылка] .

    Написать расширенную матрицу для системы уравнений

    Решение системы уравнений может быть утомительной операцией, где простая ошибка может нанести ущерб поиску решения. Доступен альтернативный метод, использующий основные процедуры исключения, но с более простыми обозначениями. Метод предполагает использование матрицы . Матрица представляет собой прямоугольный массив чисел, расположенных в строках и столбцах.

    МАТРИЦА

    Матрица представляет собой прямоугольный массив чисел, расположенных в строках и столбцах.

    Матрица с m строк и n столбцов имеет порядок \(m\times n\). Матрица слева внизу имеет 2 строки и 3 столбца, поэтому она имеет порядок \(2\times 3\). Мы говорим, что это матрица 2 на 3.

    Каждое число в матрице называется элементом или записью в матрице.

    Мы будем использовать матрицу для представления системы линейных уравнений. Мы записываем каждое уравнение в стандартной форме, а коэффициенты переменных и константы каждого уравнения становятся строкой в ​​матрице. Тогда каждый столбец будет коэффициентом одной из переменных в системе или констант. Вертикальная черта заменяет знаки равенства. Полученную матрицу назовем расширенной матрицей системы уравнений.

    Обратите внимание, что первый столбец состоит из всех коэффициентов x , второй столбец содержит все коэффициенты y , а третий столбец содержит все константы.

    Пример \(\PageIndex{1}\)

    ⓐ \(\left\{ \begin{array} {l} 5x−3y=−1 \\ y=2x−2 \end{array} \right. \) ⓑ \( \left\{ \begin{array} {l} 6x−5y+2z=3 \\ 2x+y−4z=5 \\ 3x−3y+z=−1 \end{array} \right .\)

    Ответить

    ⓐ Второе уравнение не в стандартной форме. Перепишем второе уравнение в стандартной форме.

    \[\begin{выровнено} y=2x−2 \\ −2x+y=−2 \end{выровнено} \nonumber\]

    Заменим второе уравнение его стандартной формой. В расширенной матрице первое уравнение дает нам первую строку, а второе уравнение дает нам вторую строку. Вертикальная черта заменяет знаки равенства.

    ⓑ Все три уравнения в стандартной форме. В расширенной матрице первое уравнение дает нам первую строку, второе уравнение дает нам вторую строку, а третье уравнение дает нам третью строку. Вертикальная черта заменяет знаки равенства.

    Пример \(\PageIndex{2}\)

    Запишите каждую систему линейных уравнений в виде расширенной матрицы:

    ⓐ \(\left\{ \begin{array} {l} 3x+8y=−3 \\ 2x=−5y−3 \end{array} \right. \) ⓑ \(\left\{ \begin{array} {l} 2x−5y+3z=8 \\ 3x−y+4z=7 \\ x +3y+2z=−3 \end{массив} \right.\)

    Ответ

    ⓐ \(\left[ \begin{matrix} 3 &8 &-3 \\ 2 &5 &−3 \end{matrix} \right] \)

    ⓑ \(\left[ \begin{matrix} 2 &3 &1 &−5 \\ ​​−1 &3 &3 &4 &4 \\ 2 &8 &7 &−3 \end{matrix} \right] \)

    Пример \(\PageIndex{3}\)

    Запишите каждую систему линейных уравнений в виде расширенной матрицы:

    ⓐ \(\left\{ \begin{array} {l} 11x=−9y−5 \\ ​​7x +5y=−1 \end{array} \right. \) ⓑ \(\left\{ \begin{array} {l} 5x−3y+2z=−5 \\ ​​2x−y−z=4 \\ 3x −2y+2z=−7 \end{массив} \right.\)

    Ответ

    ⓐ \(\left[ \begin{matrix} 11 &9 &−5 \\ ​​7 &5 &−1 \end{matrix} \right] \)
    ⓑ \(\left[ \begin{matrix} 5 &−3 &2 &−5 \\ ​​2 &−1 &−1 &4 \\ 3 &−2 &2 &−7 \end{matrix} \right] \)

    Когда мы решаем системы уравнений с использованием матриц, важно иметь возможность переключаться между системой и матрицей. В следующем примере нас просят взять информацию из матрицы и написать систему уравнений.

    Пример \(\PageIndex{4}\)

    Запишите систему уравнений, соответствующую расширенной матрице:

    \(\left[ \begin{array} {ccc|c} 4 &−3 &3 &− 1 \\ 1 &2 &−1 &2 \\ −2 &−1 &3 &−4 \end{массив} \right] \).

    Ответить

    Мы помним, что каждая строка соответствует уравнению и что каждая запись является коэффициентом переменной или константой. Вертикальная черта заменяет знак равенства. Поскольку эта матрица представляет собой \(4\times 3\), мы знаем, что она преобразуется в систему трех уравнений с тремя переменными.

    Пример \(\PageIndex{5}\)

    Запишите систему уравнений, соответствующую расширенной матрице: \(\left[ \begin{matrix} 1 &−1 &2 &3 \\ 2 &1 &−2 &1 \\ 4 &−1 &2 &0 \end{matrix} \right] \).

    Ответить

    \(\left\{ \begin{array} {l} x−y+2z=3 \\ 2x+y−2z=1 \\ 4x−y+2z=0 \end{array} \right.\)

    Пример \(\PageIndex{6}\)

    Запишите систему уравнений, соответствующую расширенной матрице: \(\left[ \begin{matrix} 1 &1 &1 &4 \\ 2 &3 &−1 &8 \\ 1 &1 &−1 &3 \end{matrix} \ Правильно] \).

    Ответить

    \(\left\{ \begin{array} {l} x+y+z=4 \\ 2x+3y−z=8 \\ x+y-z=3 \end{array} \right.\)

    Использование операций со строками над матрицей

    После того, как система уравнений будет представлена ​​в расширенной матричной форме, мы будем выполнять операции над строками, которые приведут нас к решению.

    Чтобы решить методом исключения, не имеет значения, в каком порядке мы располагаем уравнения в системе. Точно так же в матрице мы можем поменять местами строки.

    При решении методом исключения мы часто умножаем одно из уравнений на константу. Поскольку каждая строка представляет уравнение, и мы можем умножить каждую часть уравнения на константу, аналогичным образом мы можем умножить каждую запись в строке на любое действительное число, кроме 0.

    При исключении мы часто добавляем кратное одной строке к другой ряд. В матрице мы можем заменить строку на ее сумму, кратную другой строке.

    Эти действия называются операциями со строками и помогут нам использовать матрицу для решения системы уравнений.

    ОПЕРАЦИИ СО СТРОКАМИ

    В матрице следующие операции могут выполняться над любой строкой, и результирующая матрица будет эквивалентна исходной матрице.

    1. Поменяйте местами любые две строки.
    2. Умножить строку на любое действительное число, кроме 0.
    3. Добавить ненулевое кратное одной строки к другой строке.

    Выполнение этих операций несложно, но все арифметические действия могут привести к ошибке. Если мы используем систему для записи операций со строками на каждом этапе, гораздо проще вернуться и проверить нашу работу.

    Мы используем заглавные буквы с нижними индексами для представления каждой строки. Затем мы показываем операцию слева от новой матрицы. Чтобы показать перестановку строк:

    Чтобы умножить строку 2 на \(−3\):

    Чтобы умножить строку 2 на \(−3\) и добавить ее к строке 1:

    Пример \ (\PageIndex{7}\)

    Выполнить указанные операции над расширенной матрицей:

    ⓐ Поменять местами строки 2 и 3.

    ⓑ Умножить строку 2 на 5.

    ⓒ Умножить строку 3 на −2−2 и прибавить к строке 1.

    \( \left[ \begin{array} {ccc|c} 6 &−5 &2 &3 \\ 2 &1 &−4 &5 \\ 3 &−3 &1 &−1 \end{массив} \right] \)

    Ответ

    ⓐ Меняем местами 2 и 3 ряды.

    ⓑ Умножаем 2-й ряд на 5.

    ⓒ Умножаем строку 3 на \(−2\) и прибавляем к строке 1.

    Пример \(\PageIndex{8}\)

    Выполнить указанные операции над расширенной матрицей:

    ⓐ Поменять местами строки 1 и 3.

    ⓑ Умножить строку 3 на 3.

    ⓒ Умножить строку 3 на 2 и прибавить к строке 2.

    \( \left[ \begin{array} {ccc|c} 5 &−2 &-2 &-2 \\ 4 &-1 &−4 &4 \\ -2 &3 &0 &−1 \end{массив} \right] \)

    Ответ

    ⓐ \( \left[ \begin{matrix} −2 &3 &0 &−2 \\ 4 &−1 &−4 &4 \\ 5 &−2 &−2 &−2 \end{matrix} \right] \ )

    ⓑ \( \left[ \begin{matrix} −2 &3 &0 &−2 \\ 4 &−1 &−4 &4 \\ 15 &−6 &−6 &−6 \end{matrix} \right] \ )

    ⓒ \( \left[ \begin{matrix} -2 &3 &0 &2 & \\ 3 &4 &-13 &-16 &-8 \\ 15 &-6 &-6 &-6 & \end{matrix} \ справа] \)

    Пример \(\PageIndex{9}\)

    Выполнить указанные операции над расширенной матрицей:

    ⓐ Поменять местами строки 1 и 2,

    ⓑ Умножить строку 1 на 2,

    ⓒ Умножить строку 2 на 3 и сложить в строку 1.

    \( \left[ \begin{array} {ccc|c} 2 &−3 &−2 &−4 \\ 4 &1 &−3 &2 \\ 5 &0 &4 &−1 \end{ массив} \справа] \)

    Ответить

    ⓐ \( \left[ \begin{matrix} 4 &1 &−3 &2 \\ 2 &−3 &−2 &−4 \\ 5 &0 &4 &−1 \end{matrix} \right] \)
    ⓑ \( \left[ \begin{matrix} 8 &2 &−6 &4 \\ 2 &−3 &−2 &−4 \\ 5 &0 &4 &−1 \end{matrix} \right] \)
    ⓒ \( \left[ \begin{matrix} 14 &−7 &−12 &−8 \\ 2 &−3 &−2 &−4 \\ 5 &0 &4 &−1 \end{matrix} \right] \)

    Теперь, когда мы попрактиковались в операциях со строками, мы рассмотрим расширенную матрицу и выясним, какую операцию мы будем использовать для достижения цели. Это именно то, что мы сделали, когда мы сделали исключение. Мы решили, на какое число умножить строку, чтобы при сложении строк исключалась переменная.

    Учитывая эту систему, что бы вы сделали, чтобы исключить x ?

    Следующий пример делает то же самое, но с матрицей.

    Пример \(\PageIndex{10}\)

    Выполните необходимую операцию со строками, чтобы первая запись в строке 2 была равна нулю в расширенной матрице: \( \left[ \begin{array} {cc|c} 1 &−1 &2 \\ 4 &−8 &0 \end{массив} \right] \)

    Ответ

    Чтобы число 4 стало равным 0, мы могли бы умножить строку 1 на \(−4\), а затем прибавить ее к строке 2.

    Пример \(\PageIndex{11}\)

    Выполните необходимую операцию со строками, чтобы первая запись в строке 2 была равна нулю в расширенной матрице: \( \left[ \begin{array} {cc|c} 1 &−1 &2 \\ 3 &−6 &2 \end{массив} \right] \)

    Ответить

    \( \left[ \begin{matrix} 1 &−1 &2 \\ 0 &−3 &−4 \end{matrix} \right] \)

    Пример \(\PageIndex{12}\)

    Выполните необходимую операцию со строками, чтобы первая запись в строке 2 была равна нулю в расширенной матрице: \( \left[ \begin{array} {cc|c} 1 &−1 &3 \\ -2 &−3 &2 \end{массив} \right] \)

    Ответ

    \( \left[ \begin{matrix} 1 &−1 &3 \\ 0 &−5 &8 \end{matrix} \right] \)

    Решение системы уравнений с использованием матриц

    Чтобы решить систему уравнений с использованием матриц, мы преобразуем расширенную матрицу в матрицу строк-ступенчатой ​​формы , используя операции со строками. Для непротиворечивой и независимой системы уравнений ее расширенная матрица имеет форму эшелона строк, когда слева от вертикальной линии каждая запись на диагонали равна 1, а все записи ниже диагонали – нули.

    РЯД-ЭШЕЛОН ФОРМА

    Для непротиворечивой и независимой системы уравнений ее расширенная матрица имеет вид строк-ступенчатой ​​формы , когда слева от вертикальной линии каждая запись на диагонали равна 1, а все записи ниже диагонали – нулями.

    Как только мы приведем расширенную матрицу к ступенчатой ​​форме, мы можем написать эквивалентную систему уравнений и прочитать значение по крайней мере одной переменной. Затем мы подставляем это значение в другое уравнение, чтобы продолжить решение для других переменных. Этот процесс проиллюстрирован в следующем примере.

    Как решить систему уравнений с помощью матрицы

    Решить систему уравнений с помощью матрицы: \(\left\{ \begin{array} {l} 3x+4y=5 \\ x+2y=1 \ end{массив} \right. \)

    Ответ

    Пример \(\PageIndex{14}\)

    Решить систему уравнений с помощью матрицы: \(\left\{ \begin{array} {l} 2x+y=7 \\ x−2y=6 \end {массив} \право.\)

    Ответ

    Решение: \((4,−1)\).

    Пример \(\PageIndex{15}\)

    Решите систему уравнений с помощью матрицы: \(\left\{ \begin{array} {l} 2x+y=−4 \\ x−y=−2 \end{массив} \right.\)

    Ответ

    Решение: \((−2,0)\).

    Здесь приведены шаги.

    РЕШИТЬ СИСТЕМУ УРАВНЕНИЙ С ИСПОЛЬЗОВАНИЕМ МАТРИЦ.
    1. Напишите расширенную матрицу для системы уравнений.
    2. С помощью операций со строками значение записи в строке 1 столбца 1 равно 1.
    3. Используя операции со строками, получить нули в столбце 1 ниже 1.
    4. Используя операции со строками, сделайте запись в строке 2 столбца 2 равной 1.
    5. Продолжайте процесс до тех пор, пока матрица не будет иметь форму строки-эшелона.
    6. Напишите соответствующую систему уравнений.
    7. Используйте подстановку, чтобы найти оставшиеся переменные.
    8. Запишите решение в виде упорядоченной пары или тройки.
    9. Убедитесь, что решение соответствует исходным уравнениям.

    Вот изображение, показывающее порядок получения 1 и 0 в правильном положении для формы строки-эшелона.

    Мы используем ту же процедуру, когда система уравнений состоит из трех уравнений.

    Пример \(\PageIndex{16}\)

    Решите систему уравнений с помощью матрицы: \(\left\{ \begin{array} {l} 3x+8y+2z=−5 \\ ​​2x+5y −3z=0 \\ x+2y−2z=−1 \end{массив} \right.\)

    Ответить
    Write the augmented matrix for the equations. Row 1 is 3, 8, 2, minus 5. Row 2 is 2, 5, minus 3, 0. Row 3 is 1, 2, minus 2, minus 1. Interchange row 1 and 3 to get the entry in row 1, column 1 to be 1. Use operation minus 2R1 plus R2 on row 2. Use operation minus 3R1 plus R3 on row 3. Use operation minus 2R2 plus R3 on row 3. Use operation 1 upon 6 R3 on row 3. The matrix is now in row-echelon form. The corresponding system of equations is x plus 2y minus 2z equals minus 1, y plus z equals 2 and z equals minus 1. Using substitution, we get y equal to 3 and x equal to minus 9. The solution is minus 9, 3, minus 1. Check that the original equations hold true.”>
     
    Напишите расширенную матрицу для уравнений.
    Поменяйте местами строки 1 и 3, чтобы получить запись в
    строке 1, столбце 1, равной 1.
    Используя операции со строками, получить нули в столбце 1 ниже 1.
     
    Запись в строке 2 столбца 2 теперь равна 1.  
    Продолжайте процесс до тех пор, пока матрица
    не будет иметь форму строки-эшелона.
     
    Матрица теперь имеет форму строки-эшелона.
    Напишите соответствующую систему уравнений.
    Используйте подстановку, чтобы найти остальные переменные.
     
    Запишите решение в виде упорядоченной пары или тройки.
    Убедитесь, что решение соответствует исходным уравнениям. Мы оставляем вам чек.
    Пример \(\PageIndex{17}\)

    Решить систему уравнений с помощью матрицы: \(\left\{ \begin{array} {l} 2x−5y+3z=8 \\ 3x−y+4z =7 \\ x+3y+2z=−3 \end{массив} \right. \)

    Ответ

    \((6,−1,−3)\)

    Пример \(\PageIndex{18}\)

    Решите систему уравнений, используя матрицу: \(\left\{ \begin{array} {l} −3x+y+z=−4 \\ −x+ 2y−2z=1 \\ 2x−y−z=−1 \end{массив} \right.\)

    Ответить

    \((5,7,4)\)

    До сих пор мы работали с матрицами только с непротиворечивыми и независимыми системами, что означает, что они имеют ровно одно решение. Давайте теперь посмотрим, что происходит, когда мы используем матрицу для зависимой или противоречивой системы.

    Пример \(\PageIndex{19}\)

    Решите систему уравнений с помощью матрицы: \(\left\{ \begin{array} {l} x+y+3z=0 \\ x+3y+ 5z=0 \\ 2x+4z=1 \end{массив} \right. \)

    Ответить
     
    Напишите расширенную матрицу для уравнений.
    Запись в строке 1, столбце 1 равна 1.  
    Используя операции со строками, получить нули в столбце 1 ниже 1.
     
    Продолжайте процесс до тех пор, пока матрица не будет иметь форму строки-эшелона.
    Умножьте строку 2 на 2 и добавьте к строке 3.
    На данный момент у нас есть все нули слева от строки 3.  
    Напишите соответствующую систему уравнений.
    Так как \(0 \neq 1 \) мы имеем ложное утверждение. Точно так же, как когда мы решали систему, используя другие методы, это говорит нам о несогласованности системы. Нет решения.
    Пример \(\PageIndex{20}\)

    Решите систему уравнений, используя матрицу: \(\left\{ \begin{array} {l} x−2y+2z=1 \\ −2x+y− z=2 \\ x−y+z=5 \end{array} \right.\)

    Ответ

    нет решения

    Пример \(\PageIndex{21}\)

    Решить систему уравнений с помощью матрицы: \(\left\{ \begin{array} {l} 3x+4y−3z=−2 \\ −2x+3y −z=−1 \\ 2x+y−2z=6 \end{массив} \right. \)

    Ответ

    нет решения

    Последняя система была несогласованной и поэтому не имела решений. Следующий пример является зависимым и имеет бесконечно много решений.

    Пример \(\PageIndex{22}\)

    Решить систему уравнений с помощью матрицы: \(\left\{ \begin{array} {l} x−2y+3z=1 \\ x+y−3z =7 \\ 3x−4y+5z=7 \end{массив} \right. \)

    Ответ
     
    Напишите расширенную матрицу для уравнений.
    Запись в строке 1, столбце 1 равна 1.  
    Используя операции со строками, получить нули в столбце 1 ниже 1.
     
    Продолжайте процесс до тех пор, пока матрица не будет иметь форму строки-эшелона.
    Умножьте строку 2 на \(−2\) и добавьте ее к строке 3.
    На данный момент у нас есть все нули в нижней строке.  
    Напишите соответствующую систему уравнений.
    Так как \(0=0\) мы имеем истинное утверждение. Так же, как когда мы решали подстановкой, это говорит нам о том, что у нас есть зависимая система. Существует бесконечно много решений.
    Найдите y через z во втором уравнении.
    Решите первое уравнение для x через z .
    Замените \(y=2z+2\).
    Упрощение.
    Упрощение.
    Упрощение.
    Система имеет бесконечно много решений \((x,y,z)\), где \(x=z+5;\space y=2z+2;\space z\) – любое действительное число.
    Пример \(\PageIndex{23}\)

    Решить систему уравнений с помощью матрицы: \(\left\{ \begin{array} {l} x+y−z=0 \\ 2x+4y−2z =6 \\ 3x+6y−3z=9 \end{массив} \right. \)

    Ответ

    бесконечно много решений \((x,y,z)\), где \(x=z−3;\space y=3;\space z\) – любое действительное число.

    Пример \(\PageIndex{24}\)

    Решить систему уравнений с помощью матрицы: \(\left\{ \begin{array} {l} x−y−z=1 \\ −x+2y− 3z=−4 \\ 3x−2y−7z=0 \end{array} \right. \)

    Ответить

    бесконечно много решений \((x,y,z)\), где \(x=5z−2;\space y=4z−3;\space z\) — любое действительное число.

    Получите доступ к этому онлайн-ресурсу для получения дополнительных инструкций и практики с методом исключения Гаусса.

    • Исключение Гаусса

    Основные понятия

    • Матрица: Матрица представляет собой прямоугольный массив чисел, расположенных в строках и столбцах. Матрица с m строк и n столбцов имеют порядка \(m\times n\). Матрица слева внизу имеет 2 строки и 3 столбца, поэтому она имеет порядок \(2\times 3\). Мы говорим, что это матрица 2 на 3.

      Каждое число в матрице называется элементом или записью в матрице.
    • Операции со строками: В матрице следующие операции могут выполняться над любой строкой, и результирующая матрица будет эквивалентна исходной матрице.
      • Поменять местами любые два ряда
      • Умножить строку на любое действительное число, кроме 0
      • Добавить ненулевое кратное одной строки к другой строке
    • Строко-Эшелонная Форма: Для непротиворечивой и независимой системы уравнений ее расширенная матрица имеет форму ступенчато-строки, когда слева от вертикальной линии каждая запись на диагонали равна 1, а все записи ниже диагонали равны нули.
    • Как решить систему уравнений с помощью матриц.
      1. Напишите расширенную матрицу для системы уравнений.
      2. С помощью операций со строками значение записи в строке 1 столбца 1 равно 1.
      3. Используя операции со строками, получить нули в столбце 1 ниже 1.
      4. Используя операции со строками, сделайте запись в строке 2 столбца 2 равной 1.
      5. Продолжайте процесс до тех пор, пока матрица не будет иметь форму строки-эшелона.
      6. Напишите соответствующую систему уравнений.
      7. Используйте подстановку, чтобы найти оставшиеся переменные.
      8. Запишите решение в виде упорядоченной пары или тройки.
      9. Убедитесь, что решение соответствует исходным уравнениям.

    Глоссарий

    матрица
    Матрица представляет собой прямоугольный массив чисел, расположенных в строках и столбцах.
    рядно-эшелонная форма
    Матрица представляет собой ступенчатую матрицу, когда слева от вертикальной линии каждый элемент по диагонали равен 1, а все элементы под диагональю равны нулю.

    Эта страница под названием 4.6: Решение систем уравнений с использованием матриц распространяется под лицензией CC BY 4.0 и была создана, изменена и/или курирована OpenStax с использованием исходного контента, который был отредактирован в соответствии со стилем и стандартами платформы LibreTexts; подробная история редактирования доступна по запросу.

    1. Наверх
      • Была ли эта статья полезной?
      1. Тип изделия
        Раздел или страница
        Автор
        ОпенСтакс
        Лицензия
        СС BY
        Версия лицензии
        4,0
        Программа OER или Publisher
        ОпенСтакс
        Показать страницу TOC
        нет
      2. Теги
        1. источник@https://openstax. org/details/books/intermediate-алгебра-2e

      Решение систем уравнений матричным методом

      Матричный метод решения систем уравнений также известен как метод строк-эшелонов.
      Матричный метод аналогичен методу исключения, но намного чище
      , чем метод исключения.

      Решение систем уравнений с помощью матричного метода включает в себя выражение системы из
      уравнений в виде матрицы, а затем преобразование этой матрицы в то, что известно как форма эшелона строк
      .

      Ниже приведены два примера матриц в форме Row Echelon Form

      .

      Первая представляет собой матрицу 2 x 2 в форме Row Echelon, а вторая представляет собой матрицу .Матрица 3 x 3
      в форме Row Echelon.

      Выражение систем уравнений в виде матриц

      Учитывая следующую систему уравнений:

      Приведенная выше система уравнений с двумя переменными может быть выражена в виде матричной системы, поскольку
      следует за

      .

      Если мы решим приведенное выше, используя правила умножения матриц, мы должны получить
      с системой уравнений, с которой мы начали. Мы можем дополнительно изменить приведенные выше
      и скрыть матрицу, содержащую переменные. Мы не удаляем его, а просто прячем
      , чтобы сделать наши вычисления чище.

      Вышеупомянутое далее преобразуется в единую матрицу, как показано ниже

      .

      Часто вертикальная линия рисуется, чтобы указать, что самый правый столбец представляет
      записей справа от знака равенства в системе уравнений.

      То же самое можно сделать для системы уравнений с тремя переменными.

      Вышеупомянутое может быть выражено как произведение матриц в виде:

      Скрывая матрицу, содержащую переменные, мы можем выразить это как:

      Затем складываем все это в одну матрицу:

      или как

      Вышеупомянутая форма называется расширенной матрицей . В расширенной матрице
      выше мы знаем, что элементы слева представляют коэффициенты к переменным
      в системе уравнений.

      Метод приведения к форме Echelon Row

      Прежде чем читать этот раздел, вы должны взглянуть на раздел «Сокращение до формы эшелона
      » в разделе «Матрицы».

      Теперь, когда вы знаете, как свести матрицу к форме эшелонирования строк, давайте посмотрим, как применить
      алгоритм расширенных матриц, сформированных из систем уравнений.

      Пример 1:

      Найдите решение следующей системы уравнений

      Решение:

      Первый шаг состоит в том, чтобы выразить приведенную выше систему уравнений в виде расширенной матрицы.

      Далее мы маркируем строки:

      Теперь мы начнем фактически приводить матрицу к форме эшелона строк. Сначала мы меняем старший коэффициент
      первой строки на 1.

      Мы достигаем этого, умножая R 1 на -1 3 :

      Затем мы меняем коэффициент во второй строке, который находится ниже старшего коэффициента
      в первой строке. Это достигается путем умножения R 2 на -1 5
      , а затем прибавляя результат к R 1 .

      Добавление результата к R’ 1 :

      Итак, теперь наша новая матрица выглядит так:

      .

      На этом этапе мы повторно вводим переменные в строку 2, так как теперь у нас будет одно уравнение с
      переменными:

      Мы можем решить на y из уравнения выше:

      Теперь, когда у нас есть y , мы можем использовать обратную подстановку, чтобы найти x на
      , заменив y в уравнении с двумя переменными, составленном из R’ 1 :

      Поэтому решением системы уравнений является {x,y} = {2,-2}

      Пример 2:

      Найдите x, y и z в системе уравнений ниже

      Решение:

      Первый шаг — превратить систему уравнений с тремя переменными в расширенную матрицу 3×4
      .

      Далее помечаем строки матрицы:

      Поскольку в приведенной выше расширенной матрице мы не можем найти ни одной строки с единицей в качестве ведущих
      , нам не нужно выполнять операцию переключения строк. Однако нам нужно
      изменить строку 1 так, чтобы ее старший коэффициент был равен 1.

      Мы можем добиться этого, умножив строку 1 на 1 3 :

      Далее нам нужно изменить все записи ниже старшего коэффициента первой строки
      на нули.

      Для второй строки мы можем добиться этого, сначала умножив на -1 3
      и затем добавить результат в строку 1.

      Добавление результата в строку 1:

      Затем мы переходим к строке 3; здесь мы умножаем строку на -1 5
      , а затем добавляем результат к строке 1, чтобы обнулить первый элемент.

      Добавление результата в строку 1:

      Нам нужно, чтобы ведущий элемент во второй строке тоже был единицей. Мы получаем этот результат
      , умножив вторую строку на -3 2 :

      Затем мы обнуляем элемент в третьей строке под старшим коэффициентом во второй строке
      . Для этого умножаем третью строку на 5 4

      Добавление результата к строке 2:

      Наконец, мы умножаем строку 3 на -12, чтобы получить ведущий элемент третьей девятки.0005 ряд как один:

      Из приведенной выше матрицы мы находим переменные, начинающиеся с z в последней строке

      .

      Затем мы находим y, заменяя z в уравнении, образованном второй строкой:

      Наконец, мы находим x, подставляя значения y и z в уравнение, сформированное 9{-1}}=\dfrac{AdjA}{\left| \right|}$ и тогда мы решим вопрос.

      Полный пошаговый ответ:
      В вопросе указано, что мы должны решить систему уравнений, 4x + 3y + z = 16, 2x + y + 3z = 19, x + 2y + 4z = 25 с использованием матричного метода.
      Итак, сначала нам нужно преобразовать данные уравнения в матричную форму. Итак, мы можем написать это следующим образом.
      \[\left[ \begin{matrix}
         5 & 3 & 1 \\
         2 & 1 & 3 \\
         1 & 2 & 4 \\
      \end{matrix} \right]\left[ \begin{matrix}
         x \\
         y \\
         z \\
      \end{matrix} \right]=\left[ \begin{matrix}
         16 \ \
         19 \\
         25 \\
      \end{matrix} \right]\]
      Здесь \[A=\left[ \begin{matrix}
         5 & 3 & 1 \\
         2 & 1 & 3 \ \
         1 & 2 & 4 \\
      \end{matrix} \right],X=\left[ \begin{matrix}
         x \\
         y \\
         z \\
      \end{matrix} \right] \] и $B=\left[ \begin{matrix}
         16 \\
         19 \\
         25 \\
      \end{matrix} \right]$.
      Теперь найдем определитель матрицы A, то есть $\left| \право|$. Итак, мы получаем
      $A=\left| \begin{matrix}
         5 & 3 & 1 \\
         2 & 1 & 3 \\
         1 & 2 & 4 \\
      \end{matrix} \right|$
      = 5 (4 – 6) – 3 ( 8 – 3) + 1 (4 – 1)
      = 5 (-2) – 3 (5) + (3)
      = -10 – 15 + 3
      = -22
      Теперь найдем сопряжение матрицы A , то есть Adj A. Итак, пусть ${{c}_{ij}}$ являются кофакторами элементов ${{a}_{ij}}$ в $A\left[ {{a}_ {ij}} \right]$. Итак, получаем 9{3+3}}\влево| \begin{matrix}
         5 & 3 \\
         2 & 1 \\
      \end{matrix} \right|=1\left( 5-6 \right)=-1 \\
      \end{align}\]
      Следовательно, мы получаем матрицу кофакторов в виде $\left[ \begin{matrix}
         -2 & -5 & 3 \\
         -10 & 19 & -7 \\
         8 & -13 & -1 \\
      \end{matrix} \right]$
      Теперь, при транспонировании приведенной выше матрицы, мы получим $AdjA$ as,
      $\begin{align}
        & AdjA={{\left[ \begin{matrix}
         -2 & -5 & 3 \\ 9{-1}}=\dfrac{1}{-22}\left[ \begin{matrix}
         -2 & -10 & 8 \\
         -5 & 19 & -3 \\
         3 & -7 & – 1 \\
      \end{matrix} \right]$ и $B=\left[ \begin{matrix}
         16 \\
         19 \\
         25 \\
      \end{matrix} \right]$.
      Следовательно, мы можем написать
      $\left[ \begin{matrix}
         x \\
         y \\
         z \\
      \end{matrix} \right]=\dfrac{1}{-22}\left [ \begin{matrix}
         -2 & -10 & 8 \\
         -5 & 19 & -3 \\
         3 & -7 & -1 \\
      \end{matrix} \right]\left[ \begin{matrix}
         16 \\
         19 \\
         25 \\
      \end{matrix} \right]$
      Теперь мы выполним умножение двух матриц на правой стороне. Итак, мы можем написать
      \[\begin{align}
        & \left[ \begin{matrix}
         x \\
         y \\
         z \\
      \end{matrix} \right]=\dfrac{1 }{-22}\left[ \begin{matrix}
         -32-190+200 \\
         -80+361-325 \\
         48-133-25 \\
      \end{matrix} \right] \\
       & \left[ \begin{matrix}
         x \\
         y \\
         z \\
      \end{matrix} \right]=\dfrac{1}{-22}\left[ \begin{matrix}
      -22 \\
         -44 \\
         -110 \\
      \end{matrix} \right] \\
      \end{align}\]
      Теперь возьмем $\dfrac{1}{-22}$ и умножьте его на члены внутри матрицы, так как это константа, а мы знаем, что константы можно умножать на члены внутри матрицы. Итак, мы получим
      $\begin{align}
        & \left[ \begin{matrix}
         x \\
         y \\
         z \\
      \end{matrix} \right]=\left[ \begin{matrix}
         -22\times \dfrac{1}{-22} \\
         -44\ раз \dfrac{1}{-22} \\
         -110\times \dfrac{1}{-22} \\
      \end{matrix} \right] \\
       & \left[ \begin{matrix}
      x \\
         y \\
         z \\
      \end{matrix} \right]=\left[ \begin{matrix}
         1 \\
         2 \\
         5 \\
      \end{matrix} \right] \\
      \end{align}$
      Следовательно, мы получаем значения x = 1, y = 2 и z = 5.

      Примечание. Наиболее распространенная ошибка, которую допускают учащиеся в этом вопросе, заключается в том, что они не транспонируют матрицу, полученную после нахождения сомножителей матрицы A. Это приведет к получению неправильных ответов. Также при нахождении определителя матрицы А некоторые учащиеся могут ошибаться, меняя знаки. И учащиеся также должны знать, как выполнять умножение матриц, так как есть вероятность ошибок в вычислениях. На последнем шаге мы также можем взять -22 из матрицы, как показано ниже.
      \[\left[ \begin{matrix}
         x \\
         y \\
         z \\
      \end{matrix} \right]=\dfrac{1}{-22}\left[ \begin{matrix}
         -22 \\
         -44 \\
         -110 \\
      \end{matrix} \right]\]
      \[\left[ \begin{matrix}
         x \\
         y \\
         z \\
      \end{matrix} \right]=\dfrac{1}{-22}\times -22\left[ \begin{matrix}
         1 \\
         2 \\
         5 \\
      \end{matrix} \right ]\]
      \[\left[ \begin{matrix}
         x \\
         y \\
         z \\
      \end{matrix} \right]=\left[ \begin{matrix}
         1 \\
         2 \\
         5 \\
      \end{matrix} \right]\]
      Следовательно, получаем значения x = 1, y = 2 и z = 5.

      4.6 Решение систем уравнений с использованием определителей — алгебра среднего уровня 2e

      Цели обучения

      К концу этого раздела вы сможете:

      • Вычисление определителя матрицы 2×22×2
      • Вычисление определителя матрицы 3×33×3
      • Используйте правило Крамера для решения систем уравнений
      • Решение приложений с помощью определителей

      Приготовься 4.

      16

      Прежде чем приступить к работе, пройдите этот тест на готовность.

      Упрощение: 5(−2)−(−4)(1).5(−2)−(−4)(1).
      Если вы пропустили эту проблему, просмотрите пример 1.20.

      Приготовься 4.17

      Упрощение: −3(8−10)+(−2)(6−3)−4(−3−(−4)).−3(8−10)+(−2)(6−3) −4(−3−(−4)).
      Если вы пропустили эту проблему, просмотрите пример 1.19..

      Приготовься 4.18

      Упрощение: −12−8.−12−8.
      Если вы пропустили эту проблему, просмотрите пример 1.18.

      В этом разделе мы познакомимся с другим методом решения систем линейных уравнений, который называется правилом Крамера. Прежде чем мы сможем начать использовать правило, нам нужно выучить некоторые новые определения и обозначения.

      Вычисление определителя матрицы 2×22×2

      Если матрица имеет одинаковое количество строк и столбцов, мы называем ее квадратной матрицей. Каждая квадратная матрица имеет связанное с ней действительное число, называемое ее определителем. Чтобы найти определитель квадратной матрицы [abcd],[abcd], сначала запишем его как |abcd|.|abcd|. Чтобы получить действительное числовое значение определителя, мы вычитаем произведения диагоналей, как показано на рисунке.

      определитель

      Определитель любой квадратной матрицы [abcd],[abcd], где a, b, c, и d — действительные числа, равен

      |abcd|=ad−bc|abcd|=ad−bc

      Пример 4,45

      Вычислите определитель числа ⓐ [4−23−1][4−23−1] ⓑ [−3−4−20].[−3−4−20].

      Решение


      Запишите определитель.
      Вычтите произведения диагоналей.
      Упрощение.
      Упрощение.


      Запишите определитель.
      Вычтите произведения диагоналей.
      Упрощение.
      Упростить.

      Попытайся 4,89

      Оцените определитель ⓐ [5−32−4][5−32−4] ⓑ [−4−607].[−4−607].

      Попытайся 4,90

      Вычислите определитель числа ⓐ [−13−24][−13−24] ⓑ [−7−3−50].[−7−3−50].

      Вычисление определителя матрицы 3×33×3

      Чтобы вычислить определитель матрицы 3×33×3, мы должны иметь возможность вычислить минор записи в определителе. Минор записи – это определитель 2 × 22 × 2, найденный путем исключения строки и столбца в определителе 3 × 33 × 3, который содержит запись.

      Минор записи в определителе 3 × 33 × 3

      Младший элемент в определителе 3×33×3 — это определитель 2×22×2, найденный удалением строки и столбца в определителе 3×33×3, который содержит элемент.

      Чтобы найти минор элемента a1,a1, мы удаляем строку и столбец, которые его содержат. Таким образом, мы исключаем первую строку и первый столбец. Затем запишем оставшийся определитель 2×22×2.

      Чтобы найти минор элемента b2,b2, мы удаляем строку и столбец, которые его содержат. Таким образом, мы исключаем 2 ряд и 2 столбец. Затем запишем оставшийся определитель 2×22×2.

      Пример 4,46

      Для определителя |4−2310−3−2−42|,|4−2310−3−2−42| найдите и оцените минор числа ⓐ a1a1 ⓑ b3b3 ⓒ c2.c2.

      Решение


      Удалите строку и столбец, содержащие a1.a1.
      Запишите оставшийся определитель 2×22×2.
      Оценить.
      Упрощение.


      Удалите строку и столбец, содержащие b3.b3.
      Запишите оставшийся определитель 2×22×2.
      Оценить.
      Упрощение.


      Удалите строку и столбец, содержащие c2. c2.
      Запишите оставшийся определитель 2×22×2.
      Оценить.
      Упрощение.

      Попытайся 4,91

      Для определителя |1−1402−1−2−33|,|1−1402−1−2−33| найдите и оцените минор числа ⓐ a1a1 ⓑ b2b2 ⓒ c3.c3.

      Попытайся 4,92

      Для определителя |−2−1030−1−1−23|,|−2−1030−1−1−23| найдите и оцените минор числа ⓐ a2a2 ⓑ b3b3 ⓒ c2.c2.

      Теперь мы готовы вычислить определитель 3×33×3. Для этого мы расширимся по минорам, что позволит нам вычислить определитель 3×33×3 с помощью определителей 2×22×2, которые мы уже умеем вычислять!

      Чтобы вычислить определитель 3×33×3 путем расширения по младшим по первой строке, мы используем следующий шаблон:

      Помните, чтобы найти минор записи, мы удаляем строку и столбец, содержащие запись.

      Расширение по младшим по первой строке для оценки определителя 3 × 33 × 3

      Чтобы вычислить определитель 3×33×3 с помощью расширения по младшим по первой строке , используется следующий шаблон:

      Пример 4,47

      Вычислить определитель |2−3−1320−1−1−2||2−3−1320−1−1−2| путем расширения минорами по первой строке.

      Решение
      Расширить по минорам по первой строке
      Оцените каждый определитель.
      Упрощение.
      Упрощение.
      Упрощение.

      Попытайся 4,93

      Вычислите определитель |3−240−1−223−1|,|3−240−1−223−1|, разложив его по младшим по первой строке.

      Попытайся 4,94

      Вычислите определитель |3−2−22−14−10−3|,|3−2−22−14−10−3|, разложив его по младшим по первой строке.

      Чтобы вычислить определитель 3×33×3, мы можем разложить его по младшим, используя любую строку или столбец. Выбор строки или столбца, отличного от первой строки, иногда облегчает работу.

      Когда мы расширяем любую строку или столбец, мы должны быть осторожны со знаком членов в расширении. Чтобы определить знак терминов, мы используем следующую схему шаблона знаков.

      |+-+-+-+-+||+-+-+-+-+|

      Шаблон знака

      При расширении второстепенными элементами с использованием строки или столбца знак терминов в расширении соответствует следующему шаблону.

      |+-+-+-+-+||+-+-+-+-+|

      Обратите внимание, что шаблон знаков в первой строке соответствует знакам между терминами в расширении первой строкой.

      Поскольку мы можем расширять любую строку или столбец, как нам решить, какую строку или столбец использовать? Обычно мы пытаемся выбрать строку или столбец, которые облегчат наши вычисления. Если определитель содержит 0, использование строки или столбца, содержащего 0, упростит вычисления.

      Пример 4,48

      Вычисление определителя |4−1−33025−4−3||4−1−33025−4−3| путем расширения минорами.

      Решение

      Чтобы расширить по младшим, мы ищем строку или столбец, которые облегчат наши вычисления. Поскольку 0 находится во второй строке и втором столбце, расширение по любому из них является хорошим выбором. Поскольку во второй строке меньше отрицательных значений, чем во втором столбце, мы расширимся на вторую строку.

      Расширить второй ряд.
      Будьте осторожны со знаками.
      Оцените каждый определитель.
      Упрощение.
      Упрощение.
      Доп.

      Попытайся 4,95

      Вычислить определитель |2−1−303−43−4−3||2−1−303−43−4−3| путем расширения минорами.

      Попытайся 4,96

      Вычисление определителя |−2−1−3−1224−40||−2−1−3−1224−40| путем расширения минорами.

      Использование правила Крамера для решения систем уравнений

      Правило Крамера — это метод решения систем уравнений с использованием определителей. Его можно получить, решая общий вид системы уравнений методом исключения. Здесь мы продемонстрируем правило как для систем двух уравнений с двумя переменными, так и для систем трех уравнений с тремя переменными.

      Начнем с систем двух уравнений с двумя переменными.

      Правило Крамера для решения системы двух уравнений

      Для системы уравнений {a1x+b1y=k1a2x+b2y=k2,{a1x+b1y=k1a2x+b2y=k2 решение (x,y)(x,y) можно определить по формуле

      Обратите внимание, что для формирования определителя D мы используем коэффициенты переменных.

      Обратите внимание, что для формирования определителя DxDx и Dy,Dy мы заменяем коэффициенты переменной, которую мы находим, константами.

      Пример 4,49

      Как решить систему уравнений с помощью правила Крамера

      Решить с помощью правила Крамера: {2x+y=−43x−2y=−6.{2x+y=−43x−2y=−6.

      Решение

      Попытайся 4,97

      Решите, используя правило Крамера: {3x+y=−32x+3y=6.{3x+y=−32x+3y=6.

      Попытайся 4,98

      Решите, используя правило Крамера: {−x+y=22x+y=−4.{−x+y=22x+y=−4.

      Как

      Решить систему двух уравнений по правилу Крамера.
      1. Шаг 1. Вычислите определитель D , используя коэффициенты переменных.
      2. Шаг 2. Оцените определитель Dx.Dx. Используйте константы вместо коэффициентов x .
      3. Шаг 3. Вычислить определитель Dy.Dy. Используйте константы вместо коэффициентов y .
      4. Шаг 4. Найдите x и y . х=DxD,x=DxD,y=DyDy=DyD
      5. Шаг 5. Запишите решение в виде упорядоченной пары.
      6. Шаг 6. Убедитесь, что упорядоченная пара является решением обоих исходных уравнений.

      Чтобы решить систему трех уравнений с тремя переменными с помощью правила Крамера, мы в основном делаем то же, что и для системы двух уравнений. Однако теперь нам нужно решить для трех переменных, чтобы получить решение. Определители также будут 3×33×3, что сделает нашу работу более интересной!

      Правило Крамера для решения системы трех уравнений

      Для системы уравнений {a1x+b1y+c1z=k1a2x+b2y+c2z=k2a3x+b3y+c3z=k3,{a1x+b1y+c1z=k1a2x+b2y+c2z=k2a3x+b3y+c3z=k3 решение (x,y,z)(x,y,z) можно определить по

      Пример 4,50

      Решите систему уравнений, используя правило Крамера: {3x−5y+4z=55x+2y+z=02x+3y−2z=3. {3x−5y+4z=55x+2y+z=02x+3y−2z =3.

      Решение
      Вычислите определитель D .
      Расширить по несовершеннолетним, используя столбец 1.
      Оцените определители.
      Упростить.
      Упрощение.
      Упрощение.
      Вычислите определитель Dx.Dx. Используйте константы
      , чтобы заменить коэффициенты x .
      Расширить по несовершеннолетним, используя столбец 1.
      Оцените определители.
      Упрощение.
      Упрощение.
      Вычислите определитель Dy.Dy. Используйте константы
      , чтобы заменить коэффициенты y .
      Оцените определители.
      Упрощение.
      Упрощение.
      Упрощение.
      Вычислите определитель Dz.Dz. Используйте
      константы для замены коэффициентов z .
      Оцените определители.
      Упрощение.
      Упрощение.
      Упрощение.
      Найдите x , y и z .
      Подставьте значения.
      Упрощение.
      Запишите решение в виде упорядоченной тройки.
      Убедитесь, что упорядоченная тройка является решением
      до всех трех исходных уравнений.
      Мы оставляем чек вам.
      Решение (2,−3,−4).(2,−3,−4).

      Попытайся 4,99

      Решите систему уравнений, используя правило Крамера: {3x+8y+2z=-52x+5y-3z=0x+2y-2z=-1.{3x+8y+2z=-52x+5y-3z=0x+ 2y−2z=−1.

      Попытайся 4.100

      Решите систему уравнений, используя правило Крамера: {3x+y-6z=-32x+6y+3z=03x+2y-3z=-6.{3x+y-6z=-32x+6y+3z=03x+ 2y−3z=−6.

      Правило Крамера не работает, когда значение определителя D равно 0, так как это означало бы, что мы будем делить на 0. Но когда D=0,D=0, система либо несовместима, либо зависима.

      Когда значения D=0D=0 и Dx,DyDx,Dy и DzDz равны нулю, система непротиворечива и зависима, и существует бесконечно много решений.

      Если значения D=0D=0 и Dx,DyDx,Dy и DzDz не равны нулю, система несовместима и решения нет.

      Зависимые и несовместные системы уравнений

      Для любой системы уравнений, где значение определителя D=0,D=0,
      Значение определителейТип системыРешениеD=0иDx,DyиDzaвсе нулевыесогласованные и зависимыебесконечное число решенийD=0иDx,DyиDzaне все нулевыенесогласованныенет решенияЗначение определителейтипа системыSolutionD=0 и Dx,Dy и Dzвсе нульнепротиворечивы и зависимыбесконечно много решенийD=0иDx,DyиDzaне все нульнесовместимыотсутствуют решения

      В следующем примере мы будем использовать значения определителей, чтобы найти решение системы.

      Пример 4,51

      Решите систему уравнений, используя правило Крамера: {x+3y=4−2x−6y=3.{x+3y=4−2x−6y=3.

      Решение

      {x+3y=4−2x−6y=3Вычислить определительD, используя коэффициенты переменных. D=|13−2−6|D=−6−(−6)D=0{x+3y=4−2x −6y=3Вычислить определитель D, используя коэффициенты переменных.D=|13−2−6|D=−6−(−6)D=0

      Мы не можем использовать правило Крамера для решения этой системы. Но, взглянув на значения определителей DxDx и Dy,Dy, мы можем определить, является ли система зависимой или противоречивой.

      Вычислить определительDx.Dx=|433−6|Dx=−24−9Dx=-33Вычислить определительDx.Dx=|433−6|Dx=−24−9Dx=-33

      Поскольку все определители не равны нулю, система несовместима. Нет решения.

      Попытайся 4.101

      Решите систему уравнений, используя правило Крамера: {4x−3y=88x−6y=14.{4x−3y=88x−6y=14.

      Попытайся 4.102

      Решите систему уравнений, используя правило Крамера: {x=−3y+42x+6y=8.{x=−3y+42x+6y=8.

      Решение приложений с использованием определителей

      Интересное применение определителей позволяет нам проверить, лежат ли точки на одной прямой. Три точки (x1,y1),(x1,y1),(x2,y2)(x2,y2) и (x3,y3)(x3,y3) коллинеарны тогда и только тогда, когда определитель ниже равен нулю.

      |x1y11x2y21x3y31|=0|x1y11x2y21x3y31|=0

      Тест на коллинеарные точки

      Три точки (x1,y1),(x1,y1),(x2,y2)(x2,y2) и (x3,y3)(x3,y3) лежат на одной прямой тогда и только тогда, когда

      |x1y11x2y21x3y31|=0 |x1y11x2y21x3y31|=0

      Мы будем использовать это свойство в следующем примере.

      Пример 4,52

      Определите, лежат ли точки (5,−5),(5,−5),(4,−3),(4,−3) и (3,−1)(3,−1) на одной прямой.

      Решение
      Подставьте значения в определитель.
      (5,−5),(5,−5),(4,−3),(4,−3) и (3,−1)(3,−1)
      Оцените определитель путем расширения
      на младшие, используя столбец 3.
      Оцените определители.
      Упрощение.
      Упрощение.
      Значение определителя равно 0, поэтому
      точек лежат на одной прямой.

      Попытайся 4.103

      Определите, лежат ли точки (3,−2),(3,−2),(5,−3),(5,−3) и (1,−1)(1,−1) на одной прямой.

      Попытайся 4.104

      Определите, являются ли точки (−4,−1),(−4,−1),(−6,2),(−6,2) и (−2,−4)(−2,−4) ) коллинеарны.

      Раздел 4.6 Упражнения

      Практика ведет к совершенству

      Вычисление определителя матрицы 2 × 2

      В следующих упражнениях оцените определитель каждой квадратной матрицы.

      232.

      [6-23-1][6-23-1]

      233.

      [−48−35][−48−35]

      234.

      [−350−4][−350−4]

      235.

      [−207−5][−207−5]

      Вычисление определителя матрицы 3 × 3

      В следующих упражнениях найдите и затем оцените указанные миноры.

      236.

      |3−14−10−2−415||3−14−10−2−415|
      Найти минор ⓐ a1a1 ⓑ b2b2 ⓒ c3c3

      237.

      |−1−324−2−1−20−3||−1−324−2−1−20−3|
      Найти минор ⓐ a1a1 ⓑ b1b1 ⓒ c2c2

      238.

      |2−3−4−12−30−1−2||2−3−4−12−30−1−2|
      Найти минор ⓐ a2a2 ⓑ b2b2 ⓒ c2c2

      239.

      |−2−231−30−23−2||−2−231−30−23−2|
      Найти минор ⓐ a3a3 ⓑ b3b3 ⓒ c3c3

      В следующих упражнениях оцените каждый определитель, разложив по младшим по первой строке.

      240.

      |−23−1−12−231−3||−23−1−12−231−3|

      241.

      |4−1−2−3−21−2−57||4−1−2−3−21−2−57|

      242.

      |−2−3−45−67−120||−2−3−45−67−120|

      243.

      |13−25−640−2−1||13−25−640−2−1|

      В следующих упражнениях оцените каждый определитель, разложив его по младшим.

      244.

      |−5−1−440−32−26||−5−1−440−32−26|

      245.

      |4−133−22−104||4−133−22−104|

      246.

      |354−130−261||354−130−261|

      247.

      |2−4−35−1−4320||2−4−35−1−4320|

      Использование правила Крамера для решения систем уравнений

      В следующих упражнениях решите каждую систему уравнений, используя правило Крамера.

      248.

      {−2x+3y=3x+3y=12{−2x+3y=3x+3y=12

      249.

      {x−2y=−52x−3y=−4{x−2y=−52x−3y=−4

      250.

      {х-3у=-92х+5у=4{х-3у=-92х+5у=4

      251.

      {2x+y=-43x-2y=-6{2x+y=-43x-2y=-6

      252.

      {x−2y=−52x−3y=−4{x−2y=−52x−3y=−4

      253.

      {х-3у=-92х+5у=4{х-3у=-92х+5у=4

      254.

      {5x−3y=−12x−y=2{5x−3y=−12x−y=2

      255.

      {3x+8y=-32x+5y=-3{3x+8y=-32x+5y=-3

      256.

      {6x−5y+2z=32x+y−4z=53x−3y+z=−1{6x−5y+2z=32x+y−4z=53x−3y+z=−1

      257.

      {4x-3y+z=72x-5y-4z=33x-2y-2z=-7{4x-3y+z=72x-5y-4z=33x-2y-2z=-7

      258.

      {2x−5y+3z=83x−y+4z=7x+3y+2z=−3{2x−5y+3z=83x−y+4z=7x+3y+2z=−3

      259.

      {11x+9y+2z=-97x+5y+3z=-74x+3y+z=-3{11x+9y+2z=-97x+5y+3z=-74x+3y+z=-3

      260.

      {x+2z=04y+3z=-22x-5y=3{x+2z=04y+3z=-22x-5y=3

      261.

      {2x+5y=43y-z=34x+3z=-3{2x+5y=43y-z=34x+3z=-3

      262.

      {2y+3z=-15x+3y=-67x+z=1{2y+3z=-15x+3y=-67x+z=1

      263.

      {3x−z=−35y+2z=−64x+3y=−8{3x−z=−35y+2z=−64x+3y=−8

      264.

      {2x+y=36x+3y=9{2x+y=36x+3y=9

      265.

      {х-4у=-1-3х+12у=3{х-4у=-1-3х+12у=3

      266.

      {−3x−y=46x+2y=−16{−3x−y=46x+2y=−16

      267.

      {4x+3y=220x+15y=5{4x+3y=220x+15y=5

      268.

      {x+y-3z=-1y-z=0-x+2y=1{x+y-3z=-1y-z=0-x+2y=1

      269.

      {2x+3y+z=12x+y+z=93x+4y+2z=20{2x+3y+z=12x+y+z=93x+4y+2z=20

      270.

      {3x+4y-3z=-22x+3y-z=-12x+y-2z=6{3x+4y-3z=-22x+3y-z=-12x+y-2z=6

      271.

      {x-2y+3z=1x+y-3z=73x-4y+5z=7{x-2y+3z=1x+y-3z=73x-4y+5z=7

      Решение приложений с использованием определителей

      В следующих упражнениях определите, лежат ли заданные точки на одной прямой.

      272.

      (0,1),(0,1),(2,0),(2,0) и (-2,2).(-2,2).

      273.

      (0,−5),(0,−5),(−2,−2),(−2,−2) и (2,−8).(2,−8).

      274.

      (4,−3),(4,−3),(6,−4),(6,−4) и (2,−2).(2,−2).

      275.

      (−2,1),(−2,1),(−4,4),(−4,4) и (0,−2).(0,−2).

      Письменные упражнения

      276.

      Объясните разницу между квадратной матрицей и ее определителем. Приведите пример каждого.

      277.

      Объясните, что означает минор записи в квадратной матрице.

      278.

      Объясните, как решить, какую строку или столбец вы будете использовать для расширения определителя 3×33×3.

      279.

      Объясните шаги решения системы уравнений с помощью правила Крамера.

      Самопроверка

      ⓐ После выполнения упражнений используйте этот контрольный список, чтобы оценить свое мастерство выполнения целей этого раздела.

      ⓑ Изучив этот контрольный список, что вы сделаете, чтобы стать уверенным в выполнении всех задач?

      Как решить систему уравнений, используя обратную матрицу?

      В математике матрица представляет собой массив чисел, расположенных в виде прямоугольника и разделенных на строки и столбцы. Обычно их изображают, заключая все целые числа в квадратные скобки.

      Определитель

      Определитель матрицы — это скалярное значение, полученное для данной квадратной матрицы. Определитель рассматривается в линейной алгебре и вычисляется с использованием элементов квадратной матрицы. Определитель — это скалярное значение или число, вычисленное с использованием квадратной матрицы. Квадратная матрица может быть 2 × 2, 3 × 3, 4 × 4 или любой другой формы, в которой число столбцов и строк равно, например, n × n. Если S — множество квадратных матриц, R — множество целых чисел (действительных или комплексных) и f: S → R определяется равенством f (A) = k, где A ∈ S и k ∈ R, то f (A ) называется определителем А. Определитель изображается двумя вертикальными линиями, т. е. |A|.

      Определитель матрицы 2×2 –     

      Определитель матрицы 3×3 –  

      Миноры и кофакторы 

      Матрица, созданная после удаления строки и столбца матрицы, в которой находится этот конкретный элемент, определяется как минор матрицы.

      Минор элемента a 12 равен M 12 – 

      Кофактор элемента в матрице A получается путем умножения минора элемента M ij на (-1) i+j . C ij — это символ кофактора элемента. Если минор матрицы M ij , то кофактор элемента будет: C ij = (-1) i+j M ij . Матрица кофакторов — это матрица, созданная кофакторами компонентов матрицы.

      Матрица кофакторов:

      Сопряженная матрица

      Пусть A=[aij] — n-мерная квадратная матрица. Матрица, сопряженная с A, представляет собой транспонированную матрицу кофакторов A. Обозначается буквой adj A. Сопряженные матрицы иногда называют сопряженными матрицами. Сопряженная квадратная матрица A = [aij]n x n определяется как транспонированная матрица [Aij]n x n, где Aij — сомножитель элемента aij.

       

      Сопряженное с A = Транспонирование   = 

      Обратная матрица

      Квадратная матрица A обратима тогда и только тогда, когда A является невырожденной матрицей. Обратную матрицу можно получить, разделив сопряженную матрицу на определитель матрицы. Обратную матрицу можно вычислить, выполнив следующие шаги:

      • Шаг 1: Определите минор предоставленной матрицы.
      • Шаг 2. Преобразуйте полученную матрицу в матрицу кофакторов.
      • Шаг 3: Наконец, вспомогательное и
      • Шаг 4: Умножьте его на обратный определитель.

      Пусть A =

      Сопряженное с A = Транспонирование =

      Обратная матрица A = A {-1} =

      Применение матриц и определителей Теперь рассмотрим матричные определители и определители может использоваться для решения систем линейных уравнений с двумя или тремя переменными и для оценки непротиворечивости системы.

      • Непротиворечивая система : Система уравнений считается непротиворечивой, если она имеет (одно или несколько) решений.
      • Несовместимая система : Если решение системы уравнений не существует, говорят, что система несовместима.

      Представление линейных систем с помощью матричных уравнений

      Для представления системы уравнений можно использовать расширенную матрицу. Каждая строка в расширенной матрице представляет одно из уравнений системы, а каждый столбец представляет собой переменную или постоянные члены. Мы видим, что расширенные матрицы — это кратчайший путь для формулирования систем уравнений таким образом.

      Пример: Запишите следующую систему уравнений в виде расширенной матрицы.

      x – 2y = 5

      4x – 3y – z = 3

      5y – 7z = 9

      Представим следующую матрицу. Если переменный член не указан в матрице, считается, что коэффициент этого члена равен «0».

      (1)x + (-2)y + (0)z = 5

      (4)x + (-3)y + (-1)z = 3

      (0)x + (5)y + (-7) г = 9

      Следующая расширенная матрица:  

      Решение линейных систем с помощью матричных уравнений

      Решение линейных уравнений с использованием матрицы выполняется матричным методом. В этой статье мы рассмотрим решение линейных уравнений на матричных примерах.

      Решение уравнений с обратными матрицами

      Предположим, уравнение имеет вид:   

      Матричный метод используется для нахождения решения системы уравнений. В уравнениях все переменные должны быть записаны в правильном порядке. На соответствующих сторонах напишите переменные, их коэффициенты и константы.

      Метод определения обратного используется для решения системы линейных уравнений и требует двух дополнительных матриц. Переменные представлены матрицей X. Константы представлены матрицей B. Используя матричное умножение, система уравнений с тем же количеством уравнений, что и переменная, определяется как

      AX=B

      Пусть A будет матрица коэффициентов, X — переменная матрица, а B — постоянная матрица для решения системы линейных уравнений с обратной матрицей. В результате мы хотели бы решить систему AX = B. Взгляните на приведенные ниже уравнения в качестве примера.

      AX = B

      где:  

      Случай 1: Если A невырожденная матрица, она имеет обратную.

      Пусть A — матрица коэффициентов, X — матрица переменных, а B — матрица констант для решения системы линейных уравнений с обратной матрицей. В результате мы хотим решить систему AX=B. Чтобы получить ответ, умножьте обе части на величину, обратную A.

      Поскольку обратная матрица уникальна, это матричное уравнение предлагает единственное решение данной системы уравнений. Матричный метод — это метод решения систем уравнений.

      Случай 2: Если A — сингулярная матрица, то | А| = 0. В этом случае вычислить (adj A) B.

      Если (adj A) B ≠ O, (O — нулевые матрицы), то решения не существует и система уравнений называется несовместной.

      Если (прил. A) B = O, то система может быть либо состоятельной, либо несовместной соответственно, поскольку система либо имеет бесконечно много решений, либо не имеет решений.

      Примеры задач

      Вопрос 1: Найдите следующее из заданной матрицы  

      • determinant of matrix A
      • cofactor matrix A 
      • adjoint of matrix A
      • inverse of matrix A

      Solution:

      The given matrix is 

      • Определитель A =

      = 3(0+8)+5(-2+4)+3(-4)

      = 3 × 8 + 5 × 2 + 3 × (-4)

      = 24 + 10 – 12 единиц

      • Кофактор матрицы A =

      C 11 ​ = 0 × (-1) -4 × (-2) = 0 + 8 = 8

      C 12 ​= -((-5) × (-1) -3 × ( -2)) = -(5 + 6) = -11

      C 13 ​ = (-5) × 4 -3 × 0 = -20

      C 21 = −(2 × (-1) -4 × (-1)) = -(-2 + 4) = -2

      C 22 ​= 3 × (-1) -3 × (-1) = -3 + 3 = 0  

      C 23 ​ = -(3 × 4 – 3 × 2) = -(12 – 6) = -6

      C 31 ​ = 2 × (-2) – 0 × (-1) = -4             

      С 32 ​= -(3 × (-2) – (-5) × (-1)) = -(-6 – 5) = 11   

      C 33​ = 3 × 0 – (-5) × 2 = 10

      Кофакторная матрица a =

      • Содружка матрицы A = транспонирование кофакторной матрицы C =

      • . нанимается на работу с ежемесячной оплатой определенной суммы и ежегодным повышением на заранее определенную сумму. Найдите его начальную зарплату и ежегодную прибавку, если его зарплата составляла 300 долларов в месяц в конце первого месяца после 1 года службы и 600 долларов в месяц в конце первого месяца после 3 лет службы.

        Решение:

        Пусть «x» и «y» представляют месячную зарплату и годовое увеличение на определенную сумму соответственно.

        По вопросу;

        x + y = 300 ⇢ (i)

        x + 3y = 600 ⇢ (ii)

        Это можно записать как AX = B, где

        Определитель A = 1 × 3 – 1 × 1 = 3 – 1 = 2

        Присоединение A =

        Таким образом,

         

        Используя обратную матрицу,

        X = A -1 B

        Поэтому; x = 150 долларов, y = 150 долларов

        Итак, месячная зарплата равна 150 долларам, а годовой прирост равен 150 долларам.

        Вопрос 3: Сумма трех чисел равна 3. Если мы умножим второе число на 2 и прибавим к нему первое число, мы получим 6. Если мы умножим третье число на 4 и прибавим к нему второе число , получаем 10. Представим его алгебраически и найдем числа матричным методом.

        Решение:

        Пусть x, y и z представляют первое, второе и третье числа соответственно. Тогда согласно вопросу имеем

        x + y + z = 3

        x + 2y = 6

        y + 4z = 10

        Это можно записать как AX = B, где

        Здесь |A |= 1(8 – 0) – 1(4 – 0) + 1(1 – 0) = 8 – 4 + 1 = 5 ≠ 0. Теперь найдем прил A.

        A 11 = 8 – 0 = 8, А 12 = -(4 – 0) = -4, А 13 = 1 – 0 = 1

        А 21 = -(4 – 1) = -3, А 22 = 4 – 0 = 4, А 23 = -(1 – 0) = -1

        А 31 = 0 – 2 = -2, A 32 = -(0 – 1) = 1, A 33 = 2 – 1 = 1

        Прил. A =

        Таким образом,

        X = A -1 B

         

        Следовательно;

        Вопрос 4. Предположим, Джо, Макс и Полли пошли за покупками в торговый центр. Джо платит 45/- за 4 кг яблок, 7 кг бананов и 6 кг гуавы, Макс платит 30/- за 2 кг яблок и 5 кг гуавы, а Полли платит 35/- за 3 кг яблок, 1 кг бананов и 4 кг гуавы. Сколько стоят яблоки, бананы и гуава за килограмм?

        Решение:

        Пусть x, y и z обозначают количество яблок, бананов и гуавы соответственно.

        В соответствии с вопросом:

        4x + 7y + 6z = 45

        2 x + 5 z = 30

        3x + y + 4z = 35

        Матрица A содержит килограмм купленных яблок, бананов и гуавы Джо, Макс и Полли. Матрица B содержит цены, которые платят все трое, а матрица X содержит переменные.

        Решением данной системы уравнений будет X = A -1 B.

        Чтобы найти обратную величину A, мы сначала найдем определитель A.

        Определитель A = |A| = 4(0 х 4 – 1 х 5) – 7(2 х 4 – 5 х 3) + 6(2 х 1 – 3 х 0)

        = 4(0 – 5) – 7(8 – 15) + 6(2 – 0)

        = -20 – 7(-7) + 12

        = -20 + 49 + 12 = 41

        Прил. of A =

        Стоимость яблок за кг = 8,3/-

        Стоимость бананов за кг = 1,1/-

        Стоимость гуавы за кг = 2,7/-

        Вопрос 5: Стоимость 2 кг картофеля, 3 кг помидоров и 2 кг муки составляет 50. Стоимость 5 кг картофеля, 1 кг помидоров и 6 кг муки — 40. Стоимость 4 кг картофеля, 6 кг помидоров и 3 кг муки — 60. Найдите стоимость каждого продукта за кг с помощью обратной матрицы.

        Решение:

        Пусть x, y и z обозначают кг картофеля, помидоров и муки соответственно.

        В соответствии с вопросом:

        2x + 3y + 2z = 50

        5x + 1y + 6z = 40

        4x + 6y + 3z = 60

        Матрица A содержит кг картофеля, помидоров и муки. Матрица B содержит уплаченные цены, а матрица X содержит переменные. Это можно записать как AX = B, где  

        . Решением данной системы уравнений является X = A -1 B. Чтобы найти обратную величину A, мы сначала найдем определитель A.

      Оставить комментарий